Approche timide (mais obstinée) d'un embryon de géométrie projective - Page 2 — Les-mathematiques.net The most powerful custom community solution in the world

Approche timide (mais obstinée) d'un embryon de géométrie projective

24567

Réponses

  • Modifié (February 2022)
    (suite) 

    Considérons la collineation $\def\tra#1{{{\vphantom{#1}}^{t}{#1}}}$ $h$ qui agit sur les points $P\simeq x:y:t$ selon $P\mapsto M_{h}\cdot P$. La relation évidente: \[ \tra P\cdot\boxed{\mathcal{C}}\cdot P=\left(\tra P\cdot\tra{M_{h}}\right)\cdot\left(\tra{M_{h}}^{-1}\cdot\boxed{\mathcal{C}}\cdot M_{h}^{-1}\right)\cdot\left(M_{h}\cdot P\right) \]  montre que l'action de la collineation $M_{h}$ sur une conique $\mathcal{C}$ peut se décrire par:  \[ \widehat{h}:\boxed{\mathcal{C}}\mapsto\tra{M_{h}^{-1}}\cdot\boxed{\mathcal{C}}\cdot M_{h}^{-1} \]

    Cette action est visiblement linéaire... pour le bipède qui passe par là et qui sait que les matrices symétriques $3\times3$ forment un espace vectoriel de dimension 6. Pour rendre la chose apparente au logiciel qui devra calculer les "objets propres" , il vaut mieux décrire ces objets sous forme d'une colonne de taille 6, et de transcrire l'action $\widehat{h}$ par une matrice $6\times6$.

    C'est le moment de rappeler que faire des mathématiques, cela consiste à trouver des résultats, puis à trouver des preuves, puis à vérifier que les preuves sont probantes, et pas seulement esbroufantes. Une fois que l'on sait qu'il faut faire apparaitre des produits de droites, tout devient facile. On sait qu'une matrice dispose de colonnes propres (décrivant des points), mais aussi de lignes propres (décrivant des droites). On a donc: $\Delta_{j}\cdot M_{h}^{-1}=\lambda_{j}^{-1}\Delta_{j}$. On en déduit que:

    \[ \widehat{h}\left(\tra{\Delta_{j}}\cdot\Delta_{k}\right)=\tra{M_{h}^{-1}}\cdot\tra{\Delta_{j}}\cdot\Delta_{k}\cdot M_{h}^{-1}=\left(\lambda_{j}^{-1}\lambda_{k}^{-1}\right)\;\left(\tra{\Delta_{j}}\cdot\Delta_{k}\right) \]

    Cela nous donne une base de coniques propres. La relation $\lambda_{1}\lambda_{3}=\lambda_{2}^{2}$ fait qu'il existe un sous-espace propre de dimension $2$, engendré par $AB\times AC$ et par $BC^{2}$: c'est le faisceau bitangent dont nous n'arrêtons pas de causer.

    On remarquera que $\tra{\Delta_{j}}\cdot\Delta_{k}\neq\tra{\Delta_{k}}\cdot\Delta_{j}$ ne change rien à l'affaire: c'est la matrice symétrique: $\left(\tra{\Delta_{j}}\cdot\Delta_{k}+\tra{\Delta_{k}}\cdot\Delta_{j}\right)/2$ qui détermine la conique.  

    Cordialement, Pierre.

    Edit: les macros TeX se propagent d'un post à l'autre...  au sein d'une même page. Et lorsque l'on va "à la page", il faut les redéclarer !
  • Modifié (February 2022)
    Bonjour,
    1) Pour l'homographie $K=\dfrac{1}{12}\begin{pmatrix}18&3&6\\8&14&16\\2&2&10\end{pmatrix}$, j'essaie de présenter comme pldx1 la matrice qui révèle coniques, propres et pas propres. Il a mis les jolies valeurs propres en 1ère colonne. Elles sont effectivement en progression géométrique, et je ne vois toujours pas pourquoi.
    \[\begin{array}{c|c} 4 & \left(2x+y+2\,t\right)^{2}\\ 2 & \left(2\,x-y-2\,t\right)\left(2x+y+2\,t\right)\\ 1 & 4x^2+3y^2-24xt-12t^{2}\\ 1 & 4x^2-28xy-11y^2+88tx+28ty+100t^2\\ \dfrac{1}{2} & (y-4t)(2\,x-y-2\,t) \\ \dfrac{1}{4} & (y-4t)^2\end{array}\]
    C'est vrai que "rref" semble ne donner que des coefficients de termes carrés, ce qui permet à ceux de $xy$, $tx$ et $ty$ de bien rester planqués. Est-ce que "kernel", "nullspace", c'est du python ? Ayant bien ramé pour qu'apparaisse enfin l'hyperbole d'équation $4x^2-28xy-11y^2+88x+28y+100=0$, je suis preneur d'une simplification de ce labeur(k).
    J'ai l'impression que le message précédent contient les explications annoncées.
    "On sait qu'une matrice dispose de colonnes propres (décrivant des points), mais aussi de lignes propres (décrivant des droites)" : ça m'échappe encore, mais je m'y mets ce week-end, c'est plus que prometteur.
    2) Maintenant que des gens comme moi  admettent (faute de mieux, i.e. faute de savoir démontrer, voire de comprendre la démonstration même quand on nous la met sous le nez !) qu'on a fait le tour de toutes les coniques présentes, je continue de chercher la même hyperbole, avec des arguments d'allure plus géométriques. Je ne mets pas en doute  le fait que les matrices de pldx1 sont une quintessence géométrique, mais je dis ça parce que je ne suis pas sûr d'avoir utilisé certains renseignements sur la suspecte. a) Pour elle aussi, $(UV)$ est la polaire de $W$, b) Peut-être bien que sa matrice 3,3 est tenue d'envoyer $P$ sur $P'$ ?
    Brisons là, puisque je recommence à divaguer.
    Amicalement,
    Swingmustard
  • Bonjour !
    1) Je suis de nouveau parti de l'homographie d'une ellipse sur elle-même (cette fois : $\Gamma:x^2+3y^2=4$), définie par l'axe d'intersection des $(MP')$ avec les $(M'P)$ (j'ai pris celui d'équation $y=2$), ainsi que par $P(2,0)$ et son image $P'(1,-1)$.
    Je l'ai prolongée au plan, en une collinéation $h$. Comme l'axe ne rencontre pas l'ellipse, il y a un point fixe unique au lieu de trois dans les exemples précédents.
    Pour découvrir la formule de $h$, je n'ai pas vraiment changé de méthode pour l'instant.
    M'étant aperçu que l'homologie de centre $(a,b)$ et d'axe d'équation $cx+dy+e=0$ admet pour matrice $$M_U=\begin{pmatrix}ac-bd-e&2ad&2ae\\2bc&-ac+bd&2be\\2c&2d&-ac-bd+e\end{pmatrix}$$ je décompose $h$ en produit de deux homologies. Ici, les centres $U(0,2)$ et $V(1,2)$ et les axes qui sont leurs polaires par rapport à $\Gamma$ donnent $$H=M_V\times M_U=\begin{pmatrix}14&24&-16\\-8&12&4\\-4&-3&20\end{pmatrix}$$ J'essaie de ne pas diffuser trop vite l'image par $h$ de ce quadrilatère arbitraire et tangent deux fois, de la grille ainsi que $h(\infty)$, au cas où quelqu'un voudrait chercher. (Les sommets du quadrilatère ont des coordonnées entières, leurs images aussi.)

    2) Y a-t-il, comme pour le cas "trois points fixes", une deuxième conique propre stable ?
    Le moment était venu de programmer la diabolique matrice 6,6 de pldx1 !
    Pour un bleu en python comme moi, normal que ça ait pris du temps.
    J'ai du mal à convertir entre LateX et python: si on veut m'aider pour améliorer la présentation, ce ne sera pas de refus.

    import numpy as np
    p = np.array([[0, 0, 1],[1, 0, 0], [0, 1, 0]])
    m = np.array([[14, 24, -16], [-8, 12, 4], [-4, -3, 20]])
    % Créons quatre blocs
    a=(m*m).T
    b=2*(m*np.dot(np.dot(p,p),m)).T
    c=(m*np.dot(m,p)).T
    d=np.dot((m*np.dot(np.dot(p,m),p)+np.dot(p,m)*np.dot(m,p)).T,p)
    % Assemblons les blocs
    M=np.vstack((np.hstack((a,b)),np.hstack((c,d))))
    % Demandons valeurs et vecteurs propres
    from numpy.linalg import det
    from numpy.linalg import eig
    eig((det(m)^(2))^(-1/3)*M)
    À la sortie, j'obtiens
    \[ \dfrac{1}{18^2}\,\left[\begin{array}{rrrrrr} 196& 64& 16& -224& 64& -112\\ 576& 144& 9 & 576& -72& -144\\ 256& 400& 400& -640& 800& -640\\ 336& -96& 12& -24& -24& -138\\ -384& 240& -60& 288& 180& 528\\ -224& -160& -80& 408& -240& 344 \end{array}\right] \] 
    3) Arrivé là, eig((det(m)^(2))^(-1/3)*M) est un tableau-ligne avec les valeurs propres, couplé avec un tableau de vecteurs propres.
    J'extrais le tableau de vecteurs propres et je l'appelle V.
    Puis je tape des commandes à la "V[0,5]^-1*V[:,5]", par exemple. Celle-ci donne un multiple du vecteur correspondant à la 6ème colonne, car lignes et colonnes sont comptées de 0 à 5.
    On apprend ainsi que la valeur propre 1, double une fois de plus, correspond aux vecteurs $\begin{pmatrix}1\\4.5\\2\\0\\-3\\0\end{pmatrix}$ et $\begin{pmatrix}1\\4.78390345\\3.13561381\\0\\-3.56780691\\0\end{pmatrix}$
    On se rappelle que pldx1 a généré tous ces vecteurs en partant de matrices symétriques, la première est à peu près $$\begin{pmatrix}2&0&0\\0&9&-6\\0&-6&4\end{pmatrix}$$ Qu'est-ce que l'ensemble des points vérifiant l'équation $2x^2+9y^2-12y+4=0$ ?
    L'ellipse-point $\Omega\left(0,\dfrac{2}{3}\right)$. À vrai dire, elle était déjà indiquée comme unique point fixe sur le dessin, car les points fixes sont faciles à trouver : ce sont les vecteurs propres de la matrice 3,3, celle de $h$.
    J'ai bien peur que la recherche d'une copine pour $\Gamma$ qui, comme elle serait "conique propre invariante par $h$", échoue cette fois-ci.
    Par ailleurs, je ne sais pas reconnaître $\Gamma$ dans le deuxième vecteur !
    Amicalement,
    Swingmustard
  • Modifié (February 2022)
    Bonjour !
    Quelle image par $h$ pour le trapèze (arbitraire) voisin de l'ellipse qui, globalement, reste invariante ?

    À part ça, concernant ma recherche d'une deuxième conique que $h$ laisserait stable, j'ai seulement comme nouvelle remarque le fait que, pour les cas précédents (et je ne vois pas de raison que ce soit différent ici), le fameux axe (bleu) des intersections des $(M'P)$ et $(MP')$ est le même, que l'on considère la première ou la deuxième conique.
    Attention, c'est un axe qui n'a de sens que pour une restriction à chacune des coniques.
    Il n'existe pas pour le prolongement $h$ au plan.
    Amicalement,
    Swingmustard
  • pldx1 a dit :
    c'est la matrice symétrique: $\left(\tra{\Delta_{j}}\cdot\Delta_{k}+\tra{\Delta_{k}}\cdot\Delta_{j}\right)/2$ qui détermine la conique. 
    Je ne savais pas trop quoi mettre dans $\Delta_{j}$ et $\Delta_{k}$, mais comme j'avais deux vecteurs propres que je ne savais pas interpréter, j'ai fait comme si c'était d'eux qu'il s'agit. Je suis tombé sur une matrice qui semble constituée de deux vecteurs nuls et de quatre vecteurs grosso modo égaux. Ne sachant pas proposer mieux, je garde ce "vecteur commun" à six coordonnées.
    Il donne une matrice symétrique correspondant à l'équation $x^2+4,63y^2-3,2y+2,5=0$.
    Membre de gauche facilement strictement positif.
    Donc pas de copine réelle pour $\Gamma$ ? Va falloir qu'elle se contente d'une amie Imaginaire, peut-être bien...
    Contrairement à Alice, je vais rester de ce côté du miroir, et cesser de courir après les lapins blancs.

    Bilan du jour, par rapport au catalogue des homographies du plan chez Sidler page 38.
    I. Trois points fixes ? Vu avec des ellipses "obliques", et une ellipse "verticale" !
    II. Un point fixe et une droite stable, sur laquelle $h$ est elliptique ? On vient de voir avec cette ellipse "horizontale", et je confirme que le long de l'axe, aucun point fixe.
    III. Homologie générale ? Vu.
    IV. Homologie spéciale ? (Élation dans le reste du monde ?) Pas encore rencontrée, je crois.
    V. Deux points fixes, deux droites stables ? J'ai hâte.
    VI. Un point fixe, élément d'une droite stable, restriction parabolique à cet axe ? Connais pas.

    Concernant mon trafic pour "fabriquer" des exemples, je suis systématiquement parti d'une conique, pour la prolonger au plan. D'où une certaine obsession récente ("Où est la copine de $\Gamma$ ?"), qui devrait bientôt se calmer.

    Vu que Sidler range chaque type d'homographie suivant sa réduite de Jordan, je n'ai qu'à partir de cette réduite.
    En vue donc : IV. $\begin{pmatrix}a&0&0\\0&a&1\\0&0&a\end{pmatrix}$ pour l'homologie spéciale.
    Tiens, il propose la même matrice pour le cas V. Une coquille ? À suivre.
    Amicalement,
    Swingmustard
  • Bonjour,

    Je viens de voir ton dernier message juste au moment de poster: à suivre !

    $\def\coni{\mathcal{C}} \def\conim#1{\boxed{\coni_{#1}}}\def\ptv{~;~}$ Oui, en effet, traiter des exemples encore et encore, jusqu'à ce que la théorie sous-jacente soit bien lisse est une excellente méthode de recherche.

    On part cette fois-ci de la conique et des points de Fregier définis par:

    \[ \conim 4\simeq\left(\begin{array}{ccc} 1 & 0 & 0\\ 0 & 3 & 0\\ 0 & 0 & -4 \end{array}\right)\ptv U_{4}\simeq\left(\begin{array}{c} 0\\ 2\\ 1 \end{array}\right)\ptv V_{4}\simeq\left(\begin{array}{c} 1\\ 2\\ 1 \end{array}\right) \] L'indice $4$ veut dire qu'on en est au quatrième exemple (dans ma numérotation, arbitraire et SGDG). On en déduit  \[ \boxed{h_{4}}\doteq M_{p}\left(V_{4},\conim 4\right)\cdot M_{p}\left(U_{4},\conim 4\right)\simeq\frac{1}{18}\left[\begin{array}{ccc} 14 & 24 & -16\\ -8 & 12 & 4\\ -4 & -3 & 20 \end{array}\right] \] et la machinerie calculatoire usuelle donne (en posant $W=\sqrt{-2}$): \[ \chi_{4}=X^{3}-\dfrac{23\,X^{2}}{9}+\dfrac{23\,X}{9}-1\ptv\lambda=\dfrac{7+4W}{9},1,\dfrac{7-4W}{9}\ptv\overrightarrow{T}\simeq\left[\begin{array}{c} -2\,W\\ 2\\ 1 \end{array}\right],\left[\begin{array}{c} 0\\ 2\\ 3 \end{array}\right],\left[\begin{array}{c} 2\,W\\ 2\\ 1 \end{array}\right] \]

    Mais, si on en reste là, on risque de manquer l'essentiel, qui est: \[ \left[\begin{array}{ccc} W & 3 & -2\\ 0 & -2 & 4\\ -W & 3 & -2 \end{array}\right]\cdot\left[\begin{array}{ccc} 14 & 24 & -16\\ -8 & 12 & 4\\ -4 & -3 & 20 \end{array}\right]\cdot\left[\begin{array}{ccc} -2\,W & 0 & 2\,W\\ 2 & 2 & 2\\ 1 & 3 & 1 \end{array}\right]\simeq\left[\begin{array}{ccc} \dfrac{7+4W}{9} & 0 & 0\\ 0 & 1 & 0\\ 0 & 0 & \dfrac{7-4W}{9} \end{array}\right] \] Autrement dit, la matrice (projective) $\boxed{h_{4}}$ possède trois colonnes (projectives) propres, correspondant à trois points fixes. Mais aussi trois lignes (projectives) propres, correspondant à trois droites fixes. On rappelle que les " valeurs propres" d'une matrice projective sont définies à un facteur global près. On peut donc faire que l'une des valeurs propres (non nulles) soit $1$. En fait, c'est la proportion entre les valeurs propres qui est intrinsèque.

    La théorie ayant déjà été faite, on sait qu'une base de coniques propres pour l'opérateur $\mathcal{H}$ est formée des six produits deux à deux des trois droites propres de l'opérateur $h$. Le fait qu'il y ait des coniques non dégénérées parmi les coniques propres vient de ce que l'une des valeurs propres de $\mathcal{H}$ est double. Comme ces valeurs propres sont les produits deux à deux des valeurs propres de $h$, cela impose que $\lambda_{2}^{2}=\lambda_{1}\lambda_{3}$ (et alors les valeurs propres distinctes de $\mathcal{H}$ sont également en progression géométrique).

    Rien n'empêche de procéder " à l'aveugle" en calculant la matrice de $\mathcal{H}$, puis en calculant ses éléments propres. On trouve: \[ \begin{array}{c|c} \dfrac{17+56W}{81} & \left(-xW+2\,t-3\,y\right)^{2}\\ \dfrac{7+4W}{9} & \left(xW-2\,t+3\,y\right)\left(-y+2\,t\right)\\ 1 & x^{2}+y^{2}+8\,ty-12\,t^{2}\\ 1 & x^{2}+3\,y^{2}-4\,t^{2}\\ \dfrac{7-4W}{9} & \left(xW+2\,t-3\,y\right)\left(-y+2\,t\right)\\ \dfrac{17-56W}{81} & \left(xW+2\,t-3\,y\right)^{2} \end{array} \] On a préféré décrire $\ker\left(\mathcal{H}-1\right)$ en utilisant la conique dont on cause et un cercle comme base, plutôt que $\left(-xW+2\,t-3\,y\right)\left(+xW+2\,t-3\,y\right)$ et $\left(-y+2\,t\right)^{2}$, mais c'est juste pour faire joli: le faisceau bitangent est d'abord et avant tout engendré par $AB\times AC$ et $BC\times BC$. C'est ce qui fait marcher le barnum depuis le début.

    Remarque: le programme à la numpy est correct, mais la valeur numérique de la matrice $\boxed{\mathcal{H}}$ a été mal recopiée. Il n'en reste pas moins qu'utiliser des tiers de tour plutôt qu'une simple identification des coefficients était héroique !

    Cordialement, Pierre.
  • Bonjour à nouveau.

    $\def\crra#1#2{\operatorname{cross\_ratio}{}_{#2}\left(#1\right)} \def\tra#1{{{\vphantom{#1}}^{t}{#1}}} \def\linf{\mathcal{L}_{\infty}}$

    Voici un résumé numéroté (SGDG) des exemples utilisés dans ce fil: \[ \boxed{h_{1}}\doteq\frac{1}{21}\left[\begin{array}{ccc} 3 & -36 & -84\\ -20 & -5 & -28\\ 2 & -10 & -35 \end{array}\right]\ptv\coni_{1}\doteq x^{2}-xy+y^{2}=7\ptv\overrightarrow{F}_{1}=\left(\begin{array}{c} -2\\ -3\\ 1 \end{array}\right),\left(\begin{array}{c} 3\\ 2\\ 1 \end{array}\right)\ptv\overrightarrow{U_{1}}\doteq\left(\begin{array}{c} 1\\ 0\\ 1 \end{array}\right),\left(\begin{array}{c} 5\\ 4\\ 1 \end{array}\right) \]

    \[ \boxed{h_{2}}\doteq\frac{1}{12}\left[\begin{array}{ccc} 9 & 12 & 42\\ -5 & 28 & 70\\ -1 & 8 & 26 \end{array}\right]\ptv\coni_{2}=\coni_{1}\ptv F_{2},F_{2}'=\left(\begin{array}{c} -1\\ -3\\ 1 \end{array}\right),\left(\begin{array}{c} 2\\ 3\\ 1 \end{array}\right)\ptv U_{2},V_{2}\doteq\left(\begin{array}{c} -2\\ -7\\ 3 \end{array}\right),\left(\begin{array}{c} 0\\ -1\\ 1 \end{array}\right) \]

    \[ \boxed{h_{3}}\doteq\frac{1}{12}\left[\begin{array}{ccc} 18 & 3 & 6\\ 8 & 14 & 16\\ 2 & 2 & 10 \end{array}\right]\ptv\boxed{h_{4}}\doteq\frac{1}{18}\left[\begin{array}{ccc} 14 & 24 & -16\\ -8 & 12 & 4\\ -4 & -3 & 20 \end{array}\right] \]

    1. On remarquera que l'initiale $h$ est utilisée pour dire: ceci est une collinéation® banale, paisible et linéaire, et pas une homographie® qui serait quadratique et à la sauce Cremona. 
    2. On se donne une conique passant par $B,C$ et tangente à $AB,AC$. En barycentriques, l'équation de la conique est:  \[ \mathrm{eq}\left(A,B\right)\times\mathrm{eq}\left(A,C\right)+K\times\mathrm{eq}\left(BC\right)^{2}=Kx^{2}-yz \] On considère $U=tB+\left(1-t\right)C$. Un point $M$ de la droite $AU$ s'écrit $A+f\,U$. La condition pour que $M\left(f\right)$ soit sur la conique est $K-f^{2}\left(1-t\right)t$. Par ailleurs, on a  \[ \crra{A,U,M_{f},M_{g}}{}+1=\frac{f+g}{g} \] Or l'équation ci-dessus montre que les deux racines $f$et $g$ sont de somme nulle. Par conséquent, la division formée par $A,U$ et les deux intersections de $AU$ avec la conique est une division harmonique (que les intersections soient visibles ou non).
    3. La conique est donc invariante dans la collinéation où chaque point de $BC$ est multiplié par $+1$ et le point $A$ est multiplié par $-1$. Rem: cela est tout à fait évident sur l'équation $Kx^{2}-yz=0$.
    4.  Remplacer $\left(\begin{array}{c} x\\ y\\ t \end{array}\right)$ par la notation $\sigma\left(M\right)=\dfrac{1}{t}\left(x,y\right)$ semble être un grigri rassurant. En effet, Swingmustard semble être sur le point de flanquer par la fenêtre toutes ces salades à propos des classes d'équivalence modulo le réflexifié de la relation " sassekhoupepas" . Acte fondateur s'il en est, mais qui nécessite d'incorporer le cas $t=0$ dans la rubrique des cas normaux et inévitables.
    5. On se donne une conique, un point et sa polaire (voir exemples 1 et 2): \[ \boxed{\mathcal{C}}\simeq\left[\begin{array}{ccc} 2 & -1 & 0\\ -1 & 2 & 0\\ 0 & 0 & -14 \end{array}\right]\ptv M\simeq\left[\begin{array}{c} x\\ y\\ t \end{array}\right]\ptv\Delta\doteq\tra M\cdot\boxed{\mathcal{C}}\simeq\left[2\,x-y,-x+2\,y,-14\,t\right] \] et on écrit la procédure: \begin{eqnarray*} q_{1} & := & \mathrm{Matrix}\left(\left[\Delta\wedge\mathrm{Randomrow}(3),\Delta\wedge\mathrm{Randomrow}(3),M\right]\right) \\ q_{2} & := & \mathrm{convert} \left(\mathrm{FActor}\left(q_{1}\cdot\mathrm{Diag}(1,1,-1)\cdot q_{1}^{-1}\right),\mathrm{listlist}\right) \\ M_{p} & := & \mathrm{Matrix}\circ \mathrm{unapply}(q_{2},x,y,t)\circ OP \end{eqnarray*} \[ \dfrac{1}{x^{2}-xy+y^{2}-7\,t^{2}}\,\left[\begin{array}{ccc} y^{2}-x^{2}-7\,t^{2} & x\left(x-2\,y\right) & 14\,tx\\ -y\left(2\,x-y\right) & x^{2}-y^{2}-7\,t^{2} & 14\,ty\\ -t\left(2\,x-y\right) & t\left(x-2\,y\right) & x^{2}-xy+y^{2}+7\,t^{2} \end{array}\right] \]
    6. Remarque: il me semble qu'au moins une fois la valeur numérique de cette matrice a été remplacée par sa transposée. 
    7. On commence par vérifier que $\tra M_{p}\cdot\boxed{\mathcal{C}}\cdot M_{p}\simeq\boxed{\mathcal{C}}$. On pourrait même écrire "$=$" c'est le seul intérêt à traîner un dénominateur. 
    8. On a $\det\left(M_{u}\right)=\det\left(M_{v}\right)=-1$ et donc $\det\left(M_{v}\cdot M_{u}\right)=+1$. Posons $K\doteq\Delta_{u}\cap\Delta_{v}$. Cette colonne est multipliée deux fois par $-1$: c'est donc une colonne propre de $M_{h}$, associée à la valeur propre $+1$. La ligne propre pour la valeur $+1$ est la droite $UV$. Les deux autres valeurs propres ont $+1$ comme produit. En fait, comme $M_{h}$ est définie à un facteur (global) près, la relation invariante est $\lambda_{1}\lambda_{3}=\lambda_{2}^{2}$.
    9. Cela répond à la question: quelles sont les matrices décrivant une collinéation susceptible de fixer une conique. 
    10. Utiliser $\infty$ pour désigner la droite de l'infini n'est pas la meilleure idée de l'année. En effet, nous sommes dans un contexte où pappus utilise déjà le symbole $\infty$ pour désigner le point qui prouve qu'une droite est une droite plutôt qu'un moulin à poireaux. On peut, au contraire, utiliser la notation $\linf$ (il y a au moins une personne qui fait cela). On peut aussi se borner à décrire cette droite par ses coordonnées, qui sont $\left[0,0,1\right]$ (dans le contexte " cartésien homogène" ). 
    11. Et alors on constate que les droites sont covariantes lorsque les points sont contravariants. Jargon grandiose pour dire: pour une droite, on utilise la matrice inverse agissant à droite, et non plus la matrice directe agissant à gauche. As the joke says: les droites ne sont point des points, les points ne sont droite des droites.  
    12. Quant à arriver à la forme  \[ \frac{-1}{21}\left(\begin{array}{ccc} 49 & 0 & 0\\ 0 & -21 & 0\\ 0 & 0 & 9 \end{array}\right) \] cela s'appelle diagonaliser la matrice. On prend la base qui va bien, et alors ...grand miracle... cela se diagonalise dans la base de diagonalisation. 
    13. Remarque. Définissant les points de Fregier par $V=sF_{1}+(1-s)F_{2}\ptv U=tF_{1}+(1-t)F_{2}$ on voit aisément que: \begin{gather*} M_{u}:F_{3}\mapsto F_{3}\ptv F_{1}\mapsto F_{2}\times\left(1-1/t\right)\ptv F_{2}\mapsto F_{1}\div\left(1-1/t\right)\\ M_{v}:F_{3}\mapsto F_{3}\ptv F_{1}\mapsto F_{2}\times\left(1-1/s\right)\ptv F_{2}\mapsto F_{1}\div\left(1-1/s\right) \end{gather*} On en déduit que $M_{v}\circ M_{u}$ a pour valeurs propres $1,m,1/m$ avec $m=\frac{\left(s-1\right)t}{\left(t-1\right)s}$. L'œil du lynx clignote immédiatement, et perçoit que $m=\crra{F_{1},F_{2},U,V}{}$. Cette quantité est négative ou positive selon que $F_{1},F_{2}$ séparent $U,V$ou non (comparer exemple1 et exemple2). On a déjà rencontré cette histoire de multiplicateur lors de l'étude des prolongements par homographies®. 

    Cordialement, Pierre.
  • Modifié (February 2022)
    Bonsoir !
    1) Deux fois (ou trois fois, merci !) que tu (pldx1) dis "faisceau bitangent", et que tu sors un lapin du chapeau cercle du faisceau.
    La seule fois où je n'ai pas fait chou-blanc dans le même genre d'endroit, j'ai pris l'oseille l'hyperbole et je me suis tiré.
    Or il y a certainement, là aussi, un faisceau, dont un de ces cercles que tu exhibes avec malice !
    Ça fait d'ailleurs tout drôle de déterminer l'image de ce cercle (= lui-même) et l'image de son centre (pas lui-même du tout).
    In english : the circle centers are circlen't center circles.
    Faut que je cite Spinoza, pour prouver que je ne l'ai pas lu, et à contresens encore. "Nous ne désirons pas les choses parce qu'elles sont bonnes, mais nous les déclarons bonnes parce que nous les désirons. " Je déclare bon le faisceau parce que je le désire, et je démarre avec lui, au lieu d'échouer à le trouver.
    Posez-vous la question du plus simple faisceau bitangent de coniques qui vous vienne à l'esprit.
    Pensez-vous au même que moi, ami lecteur et belle lectrice de Robert Merle ?

    $xy=$ 1,
    $x^2+y^2=$ 2,
    $x^2+xy+y^2=$ 3, nous irons au bois,
    $x^2+2xy+y^2=$ 4, cueillir des patates (pour 5, 6, j'n'ai qu'des queues d'cerises). (Edit : J'ai terminé la comptine avec les tangentes $x+y+2=0$ et $x+y−2=0$ : fort bien. Mais tant qu'à faire, j'aurais dû la commencer avec l'axe $(x−y)^2=$ 0 !)
    Pas besoin de $M_u$ et de $M_v$ pour obtenir une formule des collinéations qui laissent invariante l'ellipse jaune, d'axe rouge et qui envoient $P$ sur $P_1$ et $P_2$ respectivement. Appelons-les $h_{5,a}(x,y,t)=(y,x,t)$ et $h_{5,c}(x,y,t)=(-x,-y,t)$, avec $a$ et $c$ comme "symétrie axiale" et "symétrie centrale".
    pldx1 dit "On a préféré décrire $ker(H-1)$ en utilisant la conique dont on cause et un cercle comme base, plutôt que $(-xW+2t-3y)(+xW+2t-3y)$ et $(-y+2t)2$, mais c'est juste pour faire joli: le faisceau bitangent est d'abord et avant tout engendré par $AB×AC$ et $BC×BC$. C'est ce qui fait marcher le barnum depuis le début."
    Enfin à moitié pigé ! Ici $A$ est à l'infini. Je n'ai pas encore compris d'où tu sors tes cercles magiques, mais on peut clairement dire que c'est dans ma caboche que ça bloque. Merci d'avoir persévéré dans le débroussaillage, malgré ma lenteur !
    Dans la classification I, II, III, IV, V, VI de Sidler mentionnée plus haut, où classeriez-vous ces deux symétries (axiale et centrale), donc ces involutions $h_{5,a}$ et $h_{5,c}$ ?
    $h_{5,a}$ semble presque trop particulière pour être de type IV = homologie spéciale, pourtant je n'arrive à la caser nulle part ailleurs. Je table sur type IV, faute de mieux.
    $h_{5,c}$ m'a tout l'air de type III = homologie générale. Son axe est simplement à l'infini, comme pour toutes les homothéties, en fait.
    2) Matrices : il est très possible que je n'aie pas encore compris qui array[[a,b],[c,d]] nous renvoie, de $\begin{pmatrix}a&b\\c&d\end{pmatrix}$ et $\begin{pmatrix}a&c\\b&d\end{pmatrix}$
    3) Tout à fait d'accord pour noter $L_\infty$ la droite de l'infini. Je n'avais pas fait exprès de partir d'un trapèze, mais ça tombe bien : les images des verticales partent se couper en un premier point de $L_\infty$, et celles des bases $(AD)$ et $(BC)$ en un deuxième.
    À ce sujet, j'aimerais qu'on me dise si les affirmations suivantes ont l'espoir de n'être pas trop délirantes.
    L'observateur se place face à 10 cm de la feuille verticale sur laquelle est tracé ceci.

    Affirmation 1. C'est en faisant pivoter la feuille autour de l'axe de $h_4$, i.e. l'horizontale passant par $B$, tout en amenant la partie basse et droite de la feuille vers lui et vers la gauche, qu'il a une chance de "voir" quelque chose qui ressemble à l'image par $h_4$ de la figure de départ, à savoir

    Affirmation 2. On pourrait calculer la position exacte à donner à la feuille pour obtenir cet effet d'optique.
    Je suis bien loin d'avoir fini d'essayer de comprendre tout ce que tu révèles, dont (et merci aussi pour) les droites droite droites (hé hé).
    Amicalement,
    Swingmustard

  • Modifié (February 2022)
    Bonjour,
    Peut-être que je pose mal ma question. Soit le trapèze $ABCD$ "affine" ci-dessus. Est-ce que le quadrilatère $A'B'C'D'$ que nous avons obtenu par la transformation $h_4$ peut, carrément, être qualifié de "vue de $ABCD$ en perspective" ?
    Argument POUR : il y a une "droite de l'infini", qu'on doit bien pouvoir rebaptiser "ligne d'horizon".
    Argument CONTRE : le terme de "perspective", au début de cette discussion, me semblait réservé aux projections centrales. Oui, mais c'était dans le cadre "d'une droite sur une droite", alors que cette question relève plutôt d'un plan qu'on envoie sur un plan, le tout en dimension 3.

    Me voilà reparti à trop mélanger. Si je me mets à croire à l'intuition de cette perspective, je crois que je vais encore passer des jours à méditer sur "Je prends $ABCD$ dans un plan, je trace $A'B'C'D'$ dans un plan sécant (par exemple $(Oy)$ puisqu'il contient $\Omega$, que $h_4$ a laissé fixe) et je cherche dans l'espace le centre d'une perspective entre les deux".

    1ère tentative (échec). J'ai changé les $x$ des points $A,B,C,D$ en des $z$, et appelé $A'',B'',C'',D''$ les points obtenus.
    Les droites $(A'A'')$ et $(C'C'')$ ont bien voulu se couper. $(B'B'')$ et $(D'D'')$ sont strictement parallèles.

    Si vous voyez tout de suite que je cherche pour rien un centre de perspective, n'hésitez pas à me ramener sur terre.
    Amicalement,
    Swingmustard
  • Modifié (February 2022)
    Bonjour !
    1) Objet d'étude : encore $ \boxed{h_{4}}\doteq \dfrac{1}{18}\left[\begin{array}{ccc} 14 & 24 & -16\\ -8 & 12 & 4\\ -4 & -3 & 20 \end{array}\right] $. Quand je dessine le faisceau que tu trouves, pldx1, et que je cherche à le qualifier, c'est (en référence aux cercles) un peu plus "à points limites" (même si je ne vois qu'un tel point) qui me vient à l'esprit, que "bitangent". Veux-tu en dire plus là-dessus ?

    2) Un sujet du début du livre de Sider, et que j'ai négligé, alors que je tourne autour constamment : le théorème de Chasles-Steiner.
    Je prends la restriction de $h_4$ à la droite $d:y=2$, autrement dit $h=\left[\begin{array}{cc} 14 & 24\times2 -16\\ -4 & -3\times2+ 20 \end{array}\right]=\left[\begin{array}{cc} 14 &32\\ -4 & 14 \end{array}\right]=\left[\begin{array}{cc} 7 &16\\ -2 & 7 \end{array}\right] $.
    (Au cas vous n'auriez pas la formule, vous sauriez la calculer à partir de trois points (par exemple $C$, $D$ et $M$) et leurs images, n'est-ce pas :) )
    Le théorème nous dit qu'étant donnés $A$ et $B$ n'appartenant pas à $d$, une bijection $h$ de $d$ sera une collinéation si et seulement si le lieu des $P\in(AM)\cap(BM')$ est une conique $\Gamma$. (Comme nous savons que la première assertion est vraie, reste à trouver $\Gamma$.)
    Même si pappus nous a familiarisés avec ce genre de relation, je trouve toujours aussi sympathiquement surprenante l'irruption d'une conique.
    Et je vous lance le défi que je n'ai, moi-même, pas encore relevé : où est-elle, cette diablesse de $\Gamma$ ?
    Peut-être son dessin vous inspirera-t-il ?

    Équation en trichant (logiciel de géométrie) : $x^2 - x y + 3y^2 + 2x + y = 6$.
    Le sens de "Où est-elle ?" est bien sûr : 1) A-t-elle une signification géométrique connue ? 2) Comment passe-t-on de l'équation de $d$, de $h$ et des coordonnées de $A$ et $B$ à l'équation de $\Gamma$ ?
    Amicalement,
    Swingmustard
  • Modifié (February 2022)
    Bonjour,
    Je recycle sur la droite $d$ une des premières fonctions rencontrées : $h(x)=\dfrac{2x+4}{x-1}$.
    (Les points $C$, $D$, $M$ et leurs images auraient pu être utilisés pour la définir.)
    Soit $A(1,1)$ et $B(1,-1)$. Le lieu des $P\in (AM)\cap(BM')$ est une conique, que je ne sais ni construire ni identifier.
    Elle nous attend pourtant là, patiemment...
    Équation de $\Gamma$ : $3x^2 - x y - 6y^2 - 7x + y + 10 = 0$, et son allure.
    Amicalement,
    Swingmustard
  • Mon cher SWingmustard
    Tu as oublié de définir la correspondance $M\mapsto M'$ sur ta figure!
    Amicalement
    pappus
  • Si si cher pappus,
    Je l'ai définie plutôt deux fois qu'une : géométriquement, elle envoie $C$, $D$, $M$ respectivement sur $C'$, $D'$, $M'$.
    Et par le calcul : à un point d'abscisse $x$, elle associe $h(x)=\dfrac{2x+4}{x-1}$, sur la même droite $d$ où elle est définie.
    Amicalement,
    Swingmustard
  • Bonjour, 

    Il est temps de mentionner qu'une collineation se décrit dans geogebra par:
    mH4 = {{14, 24, -16}, {-8, 12, 4}, {-4, -3, 20}}
    D' = ApplyMatrix[mH4, D]
    Pas besoin de tracer des dizaines de droites à coup de macros esbrouffantes. On remarquera que l'on peut passer des courbes comme argument de ApplyMatrix.

    Par contre, la gestion des matrices à coefficients complexes est difficile. Si l'on veut déterminer mH4 par le calcul, il faut en faire des tonnes.
    W = sqrt(-2+0 I )
    mat1={{W, 3, -2}, {0, -2, 4}, {-W, 3, -2}}
    mat2={{4/9*W+7/9, 0, 0}, {0, 1, 0}, {0, 0, -4/9*W+7/9}}
    mat3={{-2*W, 0, 2*W}, {2, 2, 2}, {1, 3, 1}}
    mat4=mat3*mat2*mat1*9/4
    matq= sequence(sequence( x(element(mat4,k,j)), j, 1,3),k,1,3)

    Cordialement, Pierre
  • (suite, en réponse au #2340640)

    (1) Qui sont les $\Delta_{j},\Delta_{k}$ dans $\left(\tra{\Delta_{j}}\cdot\Delta_{k}+\tra{\Delta_{k}}\cdot\Delta_{j}\right)/2$ ? Ce sont des droites. En fait, pour l'ordinateur qui fait le calcul, ce sont des vecteurs-lignes (rows)... ce qui donne un sens au produit matriciel. On a une première métonymie: droite<-->vector[row], puis une deuxième métonymie: matrice<--> conique. Et si l'on file la métonymie, il n'y a plus besoin de diviser par deux, puisque les coniques sont projectives. Bref, il y a la description explicite et contractuelle du calcul demandé au calculateur... et la perception de l'intérêt (ou du manque d'intérêt) de ce calcul.

    (2) Dans " eig((det(m)(2))(-1/3){*}M)" , j'ai l'impression qu'il s'agit de diviser $M$ par le nombre $\det\left(m\right)^{2/3}=18^{2}$. Est-ce bien le cas ?

    (3) La base " selon la théorie" de $\ker\left(\mathcal{H}_{4}-1\right)$ est $1:9/2:2:0:-3:0\ptv0:1:4:0:-2:0$. Pour faire joli, on peut prendre " notre conique" et un cercle. Les " vecteurs" proposés conviennent... mais ne vont pas faciliter le calcul numérique: le 2eme " vecteur" proposé ne contient qu'une dose de la deuxième conique pour 16 doses de la première (alors qu'on préfèrerait le contraire).

    (4) Qu'est-ce que l'ensemble des points vérifiant l'équation $2x^{2}+9y^{2}-12y+4=0$ ? L'ellipse-point $\Omega\left(0,\dfrac{2}{3}\right)$. Non certainement pas! Ce n'est pas parce que les coniques $2x^{2}+\left(3y-2t\right)^{2}=0$ et $x^{2}+\left(3y-2t\right)^{2}=0$ ne contiennent qu'un seul point visible (et le même) qu'elles sont égales ! La première est invariante sous l'action de $h$ (sur les points) et de $\mathcal{H}$ (sur les coniques). La seconde est transformée en  \[ 113\,\left(x-\frac{84\,y}{113}+\frac{56\,t}{113}\right)^{2}+\frac{59049}{113}\,\left(y-\frac{2t}{3}\right)^{2}=0 \]

    (5) Comment font les mamans chats pour reconnaître leurs petits dans un barnum pareil ? C'est un métier. On écrit que: \[ a\left[\begin{array}{ccc} 1 & 0 & 0\\ 0 & 4.78390345 & -3.56780691\\ 0 & -3.56780691 & 3.13561381 \end{array}\right]+b\left[\begin{array}{ccc} 1 & 0 & 0\\ 0 & 9/2 & -3\\ 0 & -3 & 2 \end{array}\right]-\left[\begin{array}{ccc} 1 & 0 & 0\\ 0 & 3 & 0\\ 0 & 0 & -4 \end{array}\right]\approx0 \] On appelle $\mathrm{eliminate}(\cdots,\{a,b\});$ Et la réponse est \[ \left[\left\{ a=5.28349,b=-6.28349\right\} ,\left\{ 2.6E-08,5.2E-08\right\} \right] \] Autrement dit, cela tombe dans la limite de précision des calculs, à un poil de moustache près.

    Cordialement, Pierre.
  • Modifié (February 2022)
    I. Bonjour Pierre,
    Merci pour ces renseignements sur geogebra : j'avais remarqué seul qu'ajouter 0ί était une bonne astuce pour que le logiciel se mette à travailler dans $\C$. Mais tes indications sur les matrices, dont j'avais procrastiné l'application, sont autrement instructives et prometteuses !
    En effet ApplyMatrix[mH4,(2,5,3)] alias AppliquerMatrice[mH4,(2,5,3)] applique tout-à-fait bien la matrice mH4 sur le vecteur donné, mais fonctionne aussi sur une ... fonction. Vivement que j'applique ces trouvailles.

    Tu dis "(2) Dans " eig((det(m)(2))(-1/3){*}M)" , j'ai l'impression qu'il s'agit de diviser M par le nombre det(m)2/3=182. Est-ce bien le cas ?"
    Oui, j'y ai eu recours à un moment où un blocage a dû me faire penser "Tiens, refus de puissance -2/3 pour un déterminant négatif, voyons si ça passe en modifiant un peu l'ordre des opérations".
    Mais ta question porte peut-être davantage sur "Ai-je compris pourquoi pldx1 a utilisé un dénominateur $3^2 7^2$ ?" La réponse est "Pas vraiment" : j'ai remarqué que ça t'aidait bien à "normaliser" les valeurs propres, donc j'ai copié cette action sans chercher plus loin.

    (4) D'accord. Il faut que je réfléchisse mieux à ce que signifie mon remplacement de $t$ par 1 dans les calculs (remplacement qui me rassure, comme tu l'as noté une autre fois. C'est vrai qu'avec lui, j'ai l'impression de retrouver le plancher affine des bovines).
     
    "On applique eliminate(⋯,{a,b})" : non reconnu sur ma version de geogebra. Est-ce du python ? À appliquer à l'équation matricielle au dessus ?

    II. La marotte du moment
    Au tout début (décembre) de cette discussion, j'ai appris et illustré que, dans un repère cartésien, l'hyperbole représentant $x'=\dfrac{ax+b}{cx+d}$ de mon premier souvenir de "fonctions homographiques" admet toujours un axe, lieu des  $P=(MN')\cap(M'N)$.

    Voilà pas qu'un théorème de Chasles-Steiner donne une sorte de Canada Dry de ce "lieu-droite de croisements pour une conique", puisque c'est un "lieu-conique de croisements pour une droite" (mais cette fois, $P$ est défini par $(AM)\cap(BM')$).
    Voici deux exemples plus emblématiques que les précédents.
    1) a) La droite $d$ est $(Ox)$, $h$ est définie de $d$ sur $d$ par $x'=x-2$ (Edit : la bourde d'avoir annoncé $x-1$ est corrigée le 16 février).
    On prend $A(1,1)$ et $B(-1,-1)$ (ils se doivent d'être extérieurs à $d$ ).
    b) Idem avec $x'=x+1$ (Edit : là aussi, traînait un $x'=x+\dfrac12$ erroné. Désolé !) On prend $A(1,1)$ et $B(2,\dfrac{1}2)$.
    Alors le lieu de   $P=(AM)\cap(BM')$ est ?
    L'hyperbole d'équation $xy=1$ ! Dans les deux cas.
    2) La droite $d$ demeure $(Ox)$, $h$ est définie par $x'=\dfrac{1}x$, avec $A(0,1)$ et $B(0,-1)$. Et la conique gagnante est ...
    Le cercle unité.
    Les deux cas sont si caricaturaux que les calculs de démonstration se font très vite. Mais quand on doit trouver un vecteur à six coordonnées (pour $ax^2+bxy+cy^2+dx+ey+f=0$), pldx1 a sûrement de belles matrices sous le coude (?) Il peut d'ailleurs me laisser patienter volontairement, je sens que je ferais mieux de laisser mûrir ma réflexion sur le sujet.

    À part les calculs, pappus va peut-être me montrer que j'ai la construction de ce lieu sous le nez depuis un certain temps, mais que simplement je n'ouvre pas les yeux comme il faut. (Il aura bien raison.)

    Amicalement,
    Swingmustard
  • Bonjour,

    Retour sur l'exemple 4. Ne pas confondre un faisceau bitangent avec un faisceau que l'on voit être bitangent. Le point $A_{4}$ est visible (coordonnées réelles). Les points $B_{4}$ et $C_{4}$ sont complexes conjugués. C'est pour cela qu'on ne les voit pas, et les tangentes non plus. Et, évidemment, cela perturbe quelque peu l'apparence des choses. En fait, il est particulièrement utile de contempler  la figure de cet exemple 4 comme une meilleure façon de "voir" les figures des exemples précédents.

    ------

    Appelons "exemple 5" la situation où: \[ \coni_{5}\doteq x^{2}+xy+y^{2}-3t^{2}\ptv A_{5},B_{5},C_{5}=\left(\begin{array}{c} +1\\ -1\\ 0 \end{array}\right),\left(\begin{array}{c} -1\\ -1\\ 1 \end{array}\right),\left(\begin{array}{c} +1\\ +1\\ 1 \end{array}\right)\ptv P_{5},P_{51},P_{52}\simeq\left(\begin{array}{c} 2\\ -1\\ 1 \end{array}\right),\left(\begin{array}{c} -1\\ 2\\ 1 \end{array}\right),\left(\begin{array}{c} -2\\ 1\\ 0 \end{array}\right) \]

    Les points $A,B,C$ sont les points fixes de la collinéation. Par conséquent, ils sont fixement fixes. L'involution $x:y:t\mapsto y:x:t$ est une collinéation acceptable. Par contre $x:y:t\mapsto-x:-y:t$ ne convient pas: elle permuterait les points $B,C$ au lieu de les fixer. On a montré que  \[ \boxed{h_{5}}\simeq\left[B,A,C\right]\cdot\mathrm{diag}\left(k,1,\frac{1}{k}\right)\cdot\left[B,A,C\right]^{-1}\simeq\left[\begin{array}{ccc} k+2+\dfrac{1}{k} & k-2+\dfrac{1}{k} & 2\,k-\dfrac{2}{k}\\ k-2+\dfrac{1}{k} & k+2+\dfrac{1}{k} & 2\,k-\dfrac{2}{k}\\ k-\dfrac{1}{k} & k-\dfrac{1}{k} & 2\,k+\dfrac{2}{k} \end{array}\right] \] Il reste à déterminer $k_{j}$ pour que $\boxed{h_{5}}\cdot P_{5}\simeq P_{5j}$.

    On récupère (pour $k=-1$ et $k=-1/3$): \[ \boxed{h_{51}}\simeq\left[\begin{array}{ccc} 0 & -1 & 0\\ -1 & 0 & 0\\ 0 & 0 & -1 \end{array}\right]\ptv\boxed{h_{52}}\simeq\frac{1}{3}\left[\begin{array}{ccc} -1 & -4 & 4\\ -4 & -1 & 4\\ 2 & 2 & -5 \end{array}\right] \] L'espace propre $\ker\left(H_{51}-1\right)$ est plus grand que d'habitude (dimension 4) puisque $\lambda_{3}=\lambda_{1}$. Pour ce qui est de $H_{52}$, on trouve \[ \boxed{\mathcal{H}_{5}}=\frac{1}{9}\left[\begin{array}{cccccc} 1 & 16 & 4 & 8 & -16 & -4\\ 16 & 1 & 4 & 8 & -4 & -16\\ 16 & 16 & 25 & 32 & -40 & -40\\ 4 & 4 & 4 & 17 & -10 & -10\\ -16 & -4 & -10 & -20 & 13 & 28\\ -4 & -16 & -10 & -20 & 28 & 13 \end{array}\right]\ptv\left[\begin{array}{cc} 1/9 & \left(x+y+2\,t\right)^{2}\\ -1/3 & \left(x-y\right)\left(x+y+2\,t\right)\\ 1 & \left(x+y+2\,t\right)\left(x+y-2\,t\right)\\ 1 & \left(x-y\right)^{2}\\ -3 & \left(x-y\right)\left(x+y-2\,t\right)\\ 9 & \left(x+y-2\,t\right)^{2} \end{array}\right] \]

    Il reste à se rappeler qu'une cercle est une conique qui passe par les ombilics. On substitue donc $x:y:t\mapsto1:I:0$ dans  \[ K\times\left(x+y+2\,t\right)\left(x+y-2\,t\right)+\left(x-y\right)^{2}=0 \] On résoud, on trouve $K=1$ et la " plus belle des coniques" est: $2x^{2}+2y^{2}-4t^{2}$, confirmant ce qui se voyait déjà sur la figure.

    Cordialement, Pierre
  • Modifié (February 2022)
    Merci Pierre,
    Entre la rectification de mon erreur sur $h_{52}$ et l'accompagnement/enrichissement dans ces longs messages, je réalise beaucoup te devoir.
    J'aimerais te remercier autant que pappus et le meilleur moyen serait certainement de vous montrer combien je progresse... sauf que... rire gêné, oualala... mais au diable la honte pour la lenteur !
    Suis toujours aussi ignorant de ce qu'est un ombilic (géométrie différentielle ?) mais je tomberai peut-être dessus un de ces quatre.

    Actuellement je prends deux points $A$ et $B$, une droite $d$ : disons l'axe $(Ox)$, mais notons-le plutôt $(Os)$ (je tombe sur un !), car j'aurai besoin de la lettre $x$. Sur l'axe $d=(Os)$ court un point $M(s)$ auquel une collinéation $h$ associe $M'(s')$ avec
    $s'=h(s)=\dfrac{us+v}{ws+z}$.
    Sûr à 99% que le lieu des $P\in(AM)\cap(BM')$ est une conique, j'essaie de faire une petite pause en cherchant cette conique unique (pause par rapport à la recherche de faisceau donc d'une infinité d'entre elles).
    J'en profite pour "réinvestir des acquis" récents.

    Un point $P(x,y)$ est noté, dans le contexte actuel, $P(x,y,1)$ voire $P(x,y,t)$.
    Je crois que $P(x,y,1)$ permet d'appeler le renfort des matrices, par exemple l'équation de droite $ax+by+c=0$ prend l'allure $\left(\begin{array}{c}a&b&c\end{array}\right)\left(\begin{array}{l}x\\y\\1\end{array}\right)$, et ça réserve un tas de bonnes surprises de clarté.
    Quant à $P(x,y,t)$, je crois que ce sont des coordonnées homogènes, de rigueur pour faire de la géométrie projective, même si je me cramponne souvent à $t=1$ pour "rester dans l'affine". Le cas $t=0$ nous parle bien de points à l'infini, bref.

    Une droite ? Quand Pierre insiste plaisamment (et heureusement, vu que je pige rarement du premier coup) sur le fait que dans la représentation matricielle, la droite apparaît autant sous forme de ligne que le point sous forme de colonne, ça commence à me parler très sérieusement: voir l'exemple ci-dessus, où tout nous pousse à considérer que $\left(\begin{array}{c}a&b&c\end{array}\right)$ "est" une droite, et $\left(\begin{array}{l}x\\y\\1\end{array}\right)$ (ou $\left(\begin{array}{l}x\\y\\t\end{array}\right)$) un point !

    Je cherche une conique ? Elle sera identifiée comme matrice symétrique, puisque son allure inspi(er)rée caractérisera ses points par l'équation :  $\left(\begin{array}{c}x&y&t\end{array}\right)\left(\begin{array}{c}a&d&f\\d&b&e\\f&e&c\end{array}\right)\left(\begin{array}{l}x\\y\\t\end{array}\right)=0$. Les six nombres seront arrangés en une colonne si le problème est linéaire, mais pour l'instant des $s^2$ me font penser que ce sera hors sujet ici. Fouchtra (j'ai déjà dit "bref"), je cherche six coefficients $a,b,c,d,e,f$.

    Des équations des droites $(AM)$ et $(BM')$ ? On généralise joyeusement l'équation cartésienne de droite du plan affine, il vient : $$\left(\begin{array}{c}y_A&s-x_A&-sy_A\\y_B&s'-x_B&-s'y_B\\\end{array}\right)\left(\begin{array}{l}x\\y\\t\end{array}\right)=0$$
    [Edit le 19 février à 10 heures. Pas très heureuse, cette formule : si à droite j'entérine $t$, je dois le faire à gauche aussi. Autrement dit, si on flirte avec les espaces vectoriels de dimension trois, autant le faire proprement et j'ai désormais plus confiance dans une formule comme la suivante]$$^t\left({\left(\begin{array}{c}x_B\\y_B\\t_B\end{array}\right)\wedge\left(\begin{array}{c}s'_1\\0\\s'_3\end{array}\right)}{\left(\begin{array}{c}x_A\\y_A\\t_A\end{array}\right)\wedge\left(\begin{array}{c}s_1\\0\\s_3\end{array}\right)}\right).\left(\begin{array}{l}x\\y\\t\end{array}\right)=\left(\begin{array}{l}0\\0\end{array}\right)$$
    [Ça me rappelle un déterminant, hum, pas bien réveillé.]

    Ça se résout en les inconnues $x$, $y$, $t$ sans problème (avec l'aspect homogène qui nous laisse la marge de multiplier par le dénominateur-déterminant qui nous "gênerait"), et si je remplace $s'$ par $\dfrac{us+v}{ws+z}$, il faut m'attendre à l'ensemble de points solutions (alias la conique que je cherche) $$\left(\begin{array}{l}x\\y\\t\end{array}\right)=\left(\begin{array}{l}gs^2+is+j\\ks^2+ls+m\\ns^2+ps+q\end{array}\right)$$
    Je suis à deux doigts de franchir cette étape. Prévoyant un gros blocage à venir, du style "Qu'est-ce que j'en fais ? Quelle expression les six coefficients cherchés ont-ils en fonction des neuf obtenus ?", je viens déjà demander si le calcul en vaut la peine, ou si je suis sorti de route. Éventuellement à quel virage ?

    Amicalement,
    Swingmustard

    P.S. Le rapport entre $a,b,c,d,e,f$ cherchés et $g,i,j,k,l,m,n,p,q$ en passe d'être obtenus (en fonction de $x_A,y_A,x_B,y_B,u,v,w,z$) est que, pour tout $s$ réel, on voudra avoir $$\left(\begin{array}{l}s^2&s&1\end{array}\right)\left(\begin{array}{c}g&k&n\\i&l&p\\j&m&q\end{array}\right)\left(\begin{array}{c}a&d&f\\d&b&e\\f&e&c\end{array}\right)\left(\begin{array}{c}g&i&j\\k&l&m\\n&p&q\end{array}\right)\left(\begin{array}{l}s^2\\s\\1\end{array}\right)=0$$
    Peut-être qu'il serait temps que je voie débarquer de la linéarité ?..
    En tout cas, le "Pour tout $s$ réel" légitimera l'annulation des coefficients de $s^4$, $s^3$, ... $s$, $1$ d'où cinq équations : pas si mal mais bon, j'appréhende le choc.
    Ah, mais c'est le grand retour de la "matrice héroïque" ! Posons $M=\left(\begin{array}{c}g&i&j\\k&l&m\\n&p&q\end{array}\right)$ et $P=\left(\begin{array}{c}0&1&0\\0&0&1\\1&0&0\end{array}\right)$, et notons $*$ la multiplication terme à terme de deux matrices.
    Pierre a plus que fait remarquer que le produit de trois matrices ci-dessus est symétrique. Suivant ce conseil à la lettre il y a une ou deux semaines, j'ai proposé une écriture de sa matrice $6\times6$, écriture qui n'est pas forcément la meilleure, mais fonctionne avec python. Zut, je croyais que c'était $$^t\left(\begin{array}{c}M*M&M*MP\\2M*PM&M*PMP+PM*MP\end{array}\right)\left(\begin{array}{l}a\\b\\c\\d\\e\\f\end{array}\right)$$ mais quand je relis la formule en python, je constate des différences qui signifient que j'ai sûrement modifié en dernière minute, du coup je ne dispose apparemment pas de la "bonne formule". Peu importe, l'heure est au raisonnement et à "Est-ce que, même pour cette question de conique unique, que je croyais moins ambitieuse que celle du faisceau, les formules sont si moches qu'on ne peut que les confier à l'ordi ? Et voir seulement au cas par cas des questions un peu générales du style "Quelle $h$ donne quel type de conique ?" "Hum. À suivre.
  • Modifié (February 2022)
    Bonjour !
    1) Deux exemples.
    Exemple 1.
    Soit $U(u,u^{-1})$, $V(v,v^{-1})$ deux points hors de $(Ox)$.
    Soit la transformation de $d=(Ox)$, qui envoie $M(s)$ sur $M'(s')$ par $s'=s+v-u$.
    Lieu $\Gamma$ des $P\in(UM)\cap(VM')$ ?
    J'écris ce que j'espère être les équations de $(UM)$ et de $(VM')$.
    $$\,^t\left({\left(\begin{matrix}v\\v^{-1}\\1\end{matrix}\right)\wedge\left(\begin{matrix}s+v-u\\0\\1\end{matrix}\right)}{\left(\begin{matrix}u\\u^{-1}\\1\end{matrix}\right)\wedge\left(\begin{matrix}s\\0\\1\end{matrix}\right)}\right).\left(\begin{matrix}x\\y\\t\end{matrix}\right)=\left(\begin{matrix}0\\0\end{matrix}\right)$$ Il me semble qu'ensuite, $$\left(\begin{matrix}u^{-1}&s-u&-u^{-1}s\\v^{-1}&s+v-v-u&-v^{-1}(s+v-u)\end{matrix}\right)\left(\begin{matrix}x\\y\\t\end{matrix}\right)=\left(\begin{matrix}0\\0\end{matrix}\right)$$ mènerait à $x=s-u$ et $y=\dfrac{1}{s-u}$. On pourrait en tirer directement que $\Gamma$ a pour équation $xy=1$. (Aveu déjà signé : $\Gamma$ a été fabriqué exprès, c'est d'ailleurs pour cela que $y_U$ et $y_V$ sont, d'avance, pris égaux à $u^{-1}$ et $v^{-1}$.)
    Or je ne souhaite pas trouver $\Gamma$ "trop vite" : je veux passer par la case matrices, espérant qu'elle sera, dans le cas général, plus apte à révéler la conique cherchée. $$\left(\begin{matrix}x\\y\\t\end{matrix}\right)=\left(\begin{matrix}s-u\\\dfrac{1}{s-u}\\1\end{matrix}\right)=\left(\begin{matrix}(s-u)^2\\1\\s-u\end{matrix}\right)=\left(\begin{matrix}1&-2u&u^2\\0&0&1\\0&1&-u\end{matrix}\right)\left(\begin{matrix}s^2\\s\\1\end{matrix}\right)$$ $$\left(\begin{matrix}1&0&0&0&0&0\\4u^2&0&1&0&0&-4u\\u^4&1&u^2&2u^2&-2u&-2u^3\\-2u&0&0&0&0&1\\-2u^3&0&-u&-2u&1&3u^2\\u^2&0&0&1&0&-u\end{matrix}\right)\left(\begin{matrix}a\\b\\c\\d\\e\\f\end{matrix}\right)$$ mène, me semble-t-il ("annulation des cinq coefficients des monômes en $s$", traduite par $a'=d'=b'+2f'=e'=c'=0$) à $$\left(\begin{matrix}a&d&f\\d&b&e\\f&e&c\end{matrix}\right)=\left(\begin{matrix}0&1&0\\1&0&0\\0&0&-2\end{matrix}\right)$$ i.e. $2xy-2=0$, ouf : $xy=1$.

    Exemple 2.
    Soit $U(0,1)$, $V(0,-1)$. Oui, hors de $(Ox)$.
    Soit la transformation de $d=(Ox)$, qui envoie $M(s)$ sur $M'(s')$ par $s'=\dfrac{1}{s}$.
    Lieu $\Gamma$ des $P\in(UM)\cap(VM')$ ?
    Prions pour que les équations de $(UM)$ et de $(VM')$ soient
    $$\,^t\left({\left(\begin{matrix}0\\-1\\1\end{matrix}\right)\wedge\left(\begin{matrix}s^{-1}\\0\\1\end{matrix}\right)}{\left(\begin{matrix}0\\1\\1\end{matrix}\right)\wedge\left(\begin{matrix}s\\0\\1\end{matrix}\right)}\right).\left(\begin{matrix}x\\y\\t\end{matrix}\right)=\left(\begin{matrix}0\\0\end{matrix}\right)$$ $$\left(\begin{matrix}1&s&-s\\-1&s^{-1}&s^{-1}\end{matrix}\right)\left(\begin{matrix}x\\y\\t\end{matrix}\right)=\left(\begin{matrix}0\\0\end{matrix}\right)$$ mènerait au grand classique $x=\dfrac{2s}{s^2+1}$ et $y=\dfrac{s^2-1}{s^2+1}$. Cachez ce $\Gamma$ : $x^2+y^2=1$ que je ne saurais voir.$$\left(\begin{matrix}2s\\s^2-1\\s^2+1\end{matrix}\right)=\left(\begin{matrix}0&2&0\\1&0&-1\\1&0&1\end{matrix}\right)\left(\begin{matrix}s^2\\s\\1\end{matrix}\right)$$ $$\left(\begin{matrix}0&1&1&0&2&0\\4&0&0&0&0&0\\0&1&1&0&-2&0\\0&0&0&2&0&2\\1&0&1&-2&0&2\\-1&0&1&0&0&0\end{matrix}\right)\left(\begin{matrix}a\\b\\c\\d\\e\\f\end{matrix}\right)$$ Petit détour par $a'=d'=b'+2f'=e'=c'=0$, il vient $\left(\begin{matrix}a&d&f\\d&b&e\\f&e&c\end{matrix}\right)=\left(\begin{matrix}1&0&0\\0&1&0\\0&0&-1\end{matrix}\right)$ i.e. $x^2+y^2=1$ stop !
    Votre verdict : dois-je conserver le détour par la matrice 6,6 ?
    Dit autrement : le détour le vaut-il, le détour ?
    Le vautour en vaut-il le voyage ?

    2) Relativement tôt dans le calcul, il y a une résolution de système ($x$ et $y$ sont les inconnues). Peu avant la fin : autre résolution (inconnues $a,b,c,d,e,f$). Peut-être essayer de résoudre ces systèmes en inversant des matrices ?

    La 1ère question (intérêt ou pas du détour) m'intéresse vraiment : question de raisonnement.
    La 2ème est plus anecdotique.
    Amicalement,
    Swingmustard.
  • Modifié (February 2022)
    Exemple 3 : donné le 13 février, à l'époque je n'avais aucune idée de calcul.
    Soit $U(1,1)$, $V(1,-1)$. N'appartiennent pas à $d:y=2$. On définit sur $d$ : $s'=\dfrac{2s+4}{s-1}$.
    Lieu $\Gamma$ des $P\in(UM)\cap(VM')$ ?
    Équations de $(UM)$ et de $(VM')$ ?
    $$\,^t\left({\left(\begin{matrix}1\\-1\\1\end{matrix}\right)\wedge\left(\begin{matrix}2s+4\\0\\s-1\end{matrix}\right)}{\left(\begin{matrix}1\\1\\1\end{matrix}\right)\wedge\left(\begin{matrix}s\\0\\1\end{matrix}\right)}\right).\left(\begin{matrix}x\\y\\t\end{matrix}\right)=\left(\begin{matrix}0\\0\end{matrix}\right)$$ Désencombrons la page : je supprime la fin du message, puisque l'erreur est juste là, et que le message suivant la corrige.
    Amicalement,
    Swingmustard
  • Modifié (February 2022)
    Bonjour,
    Cessons de faire concurrence à Pierre Richard, grand distrait : on a dit $y=2$, nom d'une pipe, pas $y=0$ !
    Exemple 3, deuxième prise !
    "Donné le 13 février, à l'époque je n'avais aucune idée de calcul" patati, patatras, copie copie collégram !
    Soit $U(1,1)$, $V(1,-1)$. N'appartiennent pas à $d:y=2$. On définit sur $d$ : $s'=\dfrac{2s+4}{s-1}$.
    Lieu $\Gamma$ des $P\in(UM)\cap(VM')$ ?
    Équations de $(UM)$ et de $(VM')$ ?
    $$\,^t\left({\left(\begin{matrix}1\\-1\\1\end{matrix}\right)\wedge\left(\begin{matrix}2s+4\\2s-2\\s-1\end{matrix}\right)}{\left(\begin{matrix}1\\1\\1\end{matrix}\right)\wedge\left(\begin{matrix}s\\2\\1\end{matrix}\right)}\right).\left(\begin{matrix}x\\y\\t\end{matrix}\right)=\left(\begin{matrix}0\\0\end{matrix}\right)$$ $$\left(\begin{matrix}-1&s-1&2-s\\3-3s&s+5&4s+2\end{matrix}\right)\left(\begin{matrix}x\\y\\t\end{matrix}\right)=\left(\begin{matrix}0\\0\end{matrix}\right)$$ mène à $x=\dfrac{5s^2+s-12}{3s^2-7s-2}$ et $y=\dfrac{3s^2-5s+8}{3s^2-7s-2}$.
    Au moins, là, je ne vois rien venir ! $$\left(\begin{matrix}5s^2+s-12\\3s^2-5s+8\\3s^2-7s-2\end{matrix}\right)=\left(\begin{matrix}5&1&-12\\3&-5&8\\3&-7&-2\end{matrix}\right)\left(\begin{matrix}s^2\\s\\1\end{matrix}\right)$$ Un petit coup de numpy livre $$\left(\begin{array}{c}25&9&    9&   30&   18&   30\\
              1&   25&   49&  -10&   70&  -14\\
            144&    64&   4&   -192&  -32& 48\\
              5&  -15&  -21&  -22&  -36&  -32\\
            -12&   -40&  14&   68&  -46&   82\\
            -60&   24&   -6&  4&   18&    -46\end{array}\right)\left(\begin{matrix}a\\b\\c\\d\\e\\f\end{matrix}\right)=\left(\begin{matrix}a'\\b'\\c'\\d'\\e'\\f'\end{matrix}\right)$$ Or $\left(\begin{matrix}s^2&s&1\end{matrix}\right)\left(\begin{matrix}a'&d'&f'\\d'&b'&e'\\f'&e'&c'\end{matrix}\right)\left(\begin{matrix}s^2\\s\\1\end{matrix}\right)$ n'accepte de s'annuler pour tout $s$ réel que si $a'=d'=b'+2f'=e'=c'=0$.
    Je transforme donc la ligne qui donne $b'$ en $b'+2f'$, et je supprime celle de $f'$. $$\left(\begin{array}{c}25&9&    9&   30&   18&   30\\
              -119&   73&   37&  -2&  106&  -106\\
            144&    64&   4&   -192&  -32& 48\\
              5&  -15&  -21&  -22&  -36&  -32\\
            -12&   -40&  14&   68&  -46&   82\end{array}\right)\left(\begin{matrix}a\\b\\c\\d\\e\\f\end{matrix}\right)=\left(\begin{matrix}0\\0\\0\\0\\0\end{matrix}\right)$$ Je teste le vecteur $\left(\begin{matrix}6\\-12\\20\\-1\\1\\-7\end{matrix}\right)$ qui me paraît représenter correctement la matrice attendue $\left(\begin{matrix}6&-1&-7\\-1&-12&1\\-7&1&20\end{matrix}\right)$ $$\left(\begin{array}{c}25&9&    9&   30&   18&   30\\
              -119&   73&   37&  -2&  106&  -106\\
            144&    64&   4&   -192&  -32& 48\\
              5&  -15&  -21&  -22&  -36&  -32\\
            -12&   -40&  14&   68&  -46&   82\end{array}\right)\left(\begin{matrix}6\\-12\\20\\-1\\1\\-7\end{matrix}\right)=\left(\begin{matrix}0\\0\\0\\0\\0\end{matrix}\right)$$ Ouf ! (Quelle erreur avant la modification actuelle ? J'avais interverti $y$ et $t$, relativement tôt dans le calcul.)
    Ah, j'aurais bien mérité que quelqu'un m'apprenne comment ordi ou calculatrice permettent de résoudre le système 5;6 !
    Concernant ma question "Le détour par la matrice 6,6 en vaut-il la peine ?" : pour l'instant, je ne sais pas faire plus simple.
    J'exprime à Pierre une reconnaissance qui, elle, n'est pas nuancée de "pour l'instant" : sans lui j'aurais, certes, zéro méthode de découverte de la conique cherchée, mais surtout je ne me serais pas baladé aussi agréablement dans la contrée linéaire.
    Amicalement,
    Swingmustard
  • Modifié (February 2022)
    Mon cher Swingmustard
    Je ne comprends pas tes états d'âme et tes difficultés
    Quand j'étais en taupe, je connaissais le théorème qui me disait que le lieu du point $P=UM\cap VM'$ était une conique passant par les points $U$ et $V$.
    Je n'avais pas le début du commencement de l'idée de matrices puisque nous ne faisions pas d'algèbre linéaire.
    Mais pourtant pourtant je savais écrire l'équation de ce lieu dans les délais les plus brefs c'est-à-dire en moins d'une minute.
    On pourrait faire un feuilleton à la télé intitulé:  une minute chrono!
    1° Equation de $UM$:
    $$\begin{vmatrix}
    s&2&1\\
    x&y&1\\
    1&1&1
    \end{vmatrix}
    =0
    $$
    D'où on tire:
    $$s=\dfrac{x+y-2}{y-1}$$
    2° Equation de $VM'$:
    $$\begin{vmatrix}
    s'&2&1\\
    x&y&1\\
    1&-1&1
    \end{vmatrix}
    =0
    $$
    D'où on tire:
    $$s'=\dfrac{3x+y-2}{y+1}$$
    On reporte tout ce fourbi dans la relation de liaison homographique:
    $$s'=\dfrac{2s+4}{s-1}$$
    et on tombe sur l'équation de la conique cherchée:
    $$3x^2-xy-6y^2-7x+y+10=0$$
    Amicalement
    pappus



  • Modifié (February 2022)
    Exemple 4. Dessin : donné le 12 février.
    Soit $U(1,1)$, $V(1,-1)$. N'appartiennent pas à $d:y=2$. On définit sur $d$ : $s'=\dfrac{7s+16}{-2s+7}$.
    Lieu $\Gamma$ des $P\in(UM)\cap(VM')$ ?
    Détaillons moins que l'exemple 3 qui sert de modèle.
    Équations de $(UM)$ et de $(VM')$ ? $$\,^t\left({\left(\begin{matrix}1\\-1\\1\end{matrix}\right)\wedge\left(\begin{matrix}7s+16\\-4s+14\\-2s+7\end{matrix}\right)}{\left(\begin{matrix}1\\1\\1\end{matrix}\right)\wedge\left(\begin{matrix}s\\2\\1\end{matrix}\right)}\right).\left(\begin{matrix}x\\y\\t\end{matrix}\right)=\left(\begin{matrix}0\\0\end{matrix}\right)$$ (À l'étape suivante : simplification par 3 de la deuxième ligne) $$\left(\begin{matrix}-1&s-1&2-s\\2s-7&3s+3&s+10\end{matrix}\right)\left(\begin{matrix}x\\y\\t\end{matrix}\right)=\left(\begin{matrix}0\\0\end{matrix}\right)$$ mène à $$\left(\begin{matrix}-2&-3&8\\1&-6&2\\1&-3&5\end{matrix}\right)$$ Heroic matrix $$\left(\begin{array}{c} 4&   1&   1&  -4&   2&  -4\\
            9&  36&   9&  36&  36&  18\\
    64&   4&  25&  32&  20&  80\\
      6&  -6&  -3&   9&  -9&   3\\
    -24& -12& -15& -54& -36& -39\\
    -16&   2&   5&   4&   7&  -2\end{array}\right)$$ Matrice 5,6 avec $b'+2f'$ au lieu de $b'$ et suppression de $f'$. $$\left(\begin{array}{c} 4&   1&   1&  -4&   2&  -4\\
           -23&  40&  19&  44&  50&  14\\
    64&   4&  25&  32&  20&  80\\
      6&  -6&  -3&   9&  -9&   3\\
    -24& -12& -15& -54& -36& -39\end{array}\right)$$
    Ma calculatrice prétend que cette matrice a pour réduite de Jordan l'opposé de $$\left(\begin{array}{c} -1&   0&   0&  0&   0&  1\\
           0& -1&  0&  0&  0&  3\\
    0&   0&  -1&  0&  0&  -6\\
     0&  0&  0&   -1&  0&   -0,5\\
    0& 0& 0& 0& -1& 0,5\end{array}\right)$$
    J'ignore encore ce qu'est une réduite de Jordan. Mais elle fait mon bonheur, puisqu'elle donne la matrice $$\left(\begin{matrix}1&-0,5&1\\-0,5&3&0,5\\1&0,5&-6\end{matrix}\right)$$ attendue !
    Amicalement,
    Swingmustard
    P.S. Je viens de lire ton message, pappus.
    Merci merci !
    Il est temps que je sorte, j'ai tout un tonneau de honte ... à boire jusqu'au lilas, à l'hallali, et pour lequel il me faudra plus d'une minute !!!
  • Modifié (February 2022)
    Bonsoir !
    Après la douche de limpidité infligée par pappus (encore merci, je me demande où j'ai la tête), je sauve quelques meubles dans la recherche de cette "conique de Chasles-Steiner" sous la forme d'une matrice-mystère $M=\left(\begin{array}{c}a&d&f\\d&b&e\\f&e&c\end{array}\right)$.
    On a vu qu'une condition sur $M$ est que, pour tout $s$ réel,  $$\left(\begin{array}{l}s^2&s&1\end{array}\right)\,^t N M N\left(\begin{array}{l}s^2\\s\\1\end{array}\right)=0$$
    où $N=\left(\begin{array}{c}g&k&n\\i&l&p\\j&m&q\end{array}\right)$ a valu $N_3=\left(\begin{matrix}5&1&-12\\3&-5&8\\3&-7&-2\end{matrix}\right)$ dans l'exemple 3, et $N_4=\left(\begin{matrix}-2&-3&8\\1&-6&2\\1&-3&5\end{matrix}\right)$.
    Or désormais, nous connaissons les deux matrices $M$.
    Ce sont $M_3=\left(\begin{matrix}6&-1&-7\\-1&-12&1\\-7&1&20\end{matrix}\right)$ et $M_4=\left(\begin{matrix}2&-1&2\\-1&6&1\\2&1&-12\end{matrix}\right)$.
    C'est ainsi que j'apprends ("On me cache tout, on ne me dit rien !") que la matrice $X=\,^t N M N$ qui "annule" bien tous les $\left(\begin{array}{l}s^2&s&1\end{array}\right)X\left(\begin{array}{l}s^2\\s\\1\end{array}\right)=0$, est tout simplement $X=\left(\begin{matrix}0&0&1\\0&-2&0\\1&0&0\end{matrix}\right)$, par le tour de passe-passe $s^2-2s^2+s^2=0$.
     J'aimerais d'ailleurs qu'on me dise comment interpréter cette matrice $X$.
    Moralité, renonçons dans ce problème à la matrice 6,6, et trouvons "directement" les $M$, par la simple formule $$M=(\,^t N)^{-1}X N^{-1}$$ J'admets que ça reste alambiqué : comme chez pappus, il y a un déterminant au départ, il y a ensuite une résolution de système 2,2 avec paramètre $s$ pour trouver $N$, mais si ensuite on accepte de confier le calcul d'inverse et de produit aux machines, le but est atteint.
    En largement plus d'une ... et même quelques minutes bien sûr ;)
    Amicalement,
    Swingmustard
    Moi qui me croyais sorti d'affaire... Ce n'est pas parce que mes deux exemples semblent correspondre à $$X=\left(\begin{matrix}0&0&1\\0&-2&0\\1&0&0\end{matrix}\right)$$ que ce sera toujours le cas. Qui me dit qu'on n'a pas, à l'occasion, $$X=\left(\begin{matrix}0&0&3\\0&4&0\\-7&0&0\end{matrix}\right)$$ et autres joyeusetés ? Or sans connaître $X$, la méthode tombe à l'eau ! Ah, mais par construction, $X$ est symétrique, je crois que je peux dormir sur mes deux oreilles. Bonne nuit !
  • Modifié (February 2022)
    Bonjour, 

    $\def\where{\qquad\mathrm{where}\;}$ Swingmustard fait quelques calculs et arrive à $P\doteq\left[\begin{array}{c} 5\,s^{2}+s-12\\ 3\,s^{2}-5\,s+8\\ 3\,s^{2}-7\,s-2 \end{array}\right]$. Et alors Swingmustard se demande quel est le lieu de $P(s)$. Une méthode possible est la méthode des coefficients indéterminés, c'est à dire écrire $K\doteq\tra P\cdot\boxed{m_{jk}}\cdot P=0$ ... et résoudre.

    En séparant les coefficients qui parlent d'objets différents et en les regroupant dans des structures adequates, cela donne: \[ K\doteq\tra P\cdot\boxed{m_{jk}}\cdot P=\tra{\left(\begin{array}{c} s^{4}\\ s^{3}\\ s^{2}\\ s\\ 1 \end{array}\right)\cdot\left[\begin{array}{cccccc} 25 & 9 & 9 & 30 & 18 & 30\\ 10 & -30 & -42 & -44 & -72 & -64\\ -119 & 73 & 37 & -2 & 106 & -106\\ -24 & -80 & 28 & 136 & -92 & 164\\ 144 & 64 & 4 & -192 & -32 & 48 \end{array}\right]\cdot\left[\begin{array}{c} m_{11}\\ m_{22}\\ m_{33}\\ m_{12}\\ m_{23}\\ m_{13} \end{array}\right]} \] Le fait d'obtenir une matrice $5\times6$ n'est pas le résultat d'un bricolo quelconque, c'est le résultat d'une méthode générale. Ensuite, c'est une condition nécessaire. Si l'on arrivait à une matrice $6\times6$ qui aurait la mauvaise idée d'être de rang $6$, il n'y aurait pas d'autre possibilité que $m_{jk}=0$! Comme nous avons en réalité un système de rang $5$, on peut le résoudre en prenant les cofacteurs, obtenant: \[ \tra{\left[1934917632,-3869835264,6449725440,-322486272,322486272,-2257403904\right]} \] puis en réduisant le vecteur obtenu. Cela conduit à $\overrightarrow{m}\simeq\left[6,-12,20,-1,1,-7\right]$ et donc  \[ \boxed{\mathcal{C}}\simeq\left[\begin{array}{ccc} 6 & -1 & -7\\ -1 & -12 & 1\\ -7 & 1 & 20 \end{array}\right] \]

    Remarquons que cette matrice a été trouvée et publiée par Swingmustard. Et par personne d'autre. Ensuite de quoi pappus arrive et montre que si on se posait un autre problème, cet autre problème serait plus facile à résoudre. Excellente remarque (toute publicité pour les bases de Groebner est excellente, même si cette remarque ne s'applique pas au problème posé).

    Essayons néanmoins de voir comment résoudre le problème posé (plutôt que d'en résoudre un autre). On veut trouver $\boxed{m_{jk}}$ tel que  \[ \left(\tra{\left(\begin{array}{c} s^{2}\\ s\\ 1 \end{array}\right)}\cdot\tra{\left[\begin{array}{ccc} 5 & 1 & -12\\ 3 & -5 & 8\\ 3 & -7 & -2 \end{array}\right]}\right)\cdot\boxed{m_{jk}}\cdot\left(\left[\begin{array}{ccc} 5 & 1 & -12\\ 3 & -5 & 8\\ 3 & -7 & -2 \end{array}\right]\cdot\left(\begin{array}{c} s^{2}\\ s\\ 1 \end{array}\right)\right)=0 \] L'idée stratosphérique est d'écrire cela sous la forme

    \[ \tra{\left(\begin{array}{c} s^{2}\\ s\\ 1 \end{array}\right)}\cdot\boxed{\Gamma}\cdot\left(\begin{array}{c} s^{2}\\ s\\ 1 \end{array}\right)=0\where\boxed{\Gamma}\doteq\tra P\cdot\boxed{m_{jk}}\cdot P\ptv P\doteq\left[\begin{array}{ccc} 5 & 1 & -12\\ 3 & -5 & 8\\ 3 & -7 & -2 \end{array}\right] \] En quoi cette réécriture est-elle stratosphérique ? Cela vient de $s\times s=s^{2}\times1$. Autrement dit,  \[ \boxed{\Gamma}=\left(\begin{array}{ccc} 0 & 0 & 1\\ 0 & 2 & 0\\ 1 & 0 & 0 \end{array}\right) \] est une solution. On remarquera que, dans son message #2343078, Swingmustard avait déjà indiqué que cette solution est la seule possible (sans tirer tout ce qui est possible de cette remarque) 

    On pourrait essayer d'inverser la matrice $P$. Mais ce n'est pas la méthode la plus efficace. Il vaut mieux inverser les matrices $\boxed{\Gamma}$ et $\boxed{m_{jk}}$. Autrement dit, il vaut mieux passer aux matrices décrivant les coniques tangentielles. Et on arrive à l'algorithme locusconi.



    Evidemment, cela redonne la même chose. Néanmoins, dans les situations où les coefficients font intervenir d'autres paramètres, les calculs de déterminants $5\times5$ ne coagulent pas au fur et à mesure et engendrent des expressions ayant 120 termes qui, eux mêmes, ne sont pas simples. L'avantage de locusconi est donc un (large) avantage de complexité.

    Cordialement, Pierre.

  • Modifié (February 2022)
    (suite)

    Quelques remarques en vrac: 
    ---2342093
    I-(2). Dans cet exemple, le diviseur $\def\vz{\mathrm{\mathbf{Z}}} \def\vzz{\overline{\mathcal{Z}}} \def\vt{\mathrm{\mathbf{T}}}$$3^{2}7^{2}$ permettant de ramener à la valeur $1$ le déterminant de $H$ est le carré du diviseur $3\times 7$ permettant de ramener à $1$ le déterminant de $h$.

    I-eliminate. C'est une commande Maple utilisant les bases de Groebner. Cela permet une résolution partielle d'un système d'équations. Certaines variables (ici: a,b) sont explicitées par rapport aux autres, et il reste des équations résiduelles, concernant les autres variables.

    II-marotte du moment. La relation homographique® $x'=\dfrac{ax+b}{cx+d}$ qui est utilisée est une relation entre les paramètres décrivant des points faisant partie d'une famille à un paramètre. Il n'y a pas d'hyperbole là-dedans.

    II-locusconi (encore). On sait que les " wedge" se calculent à coup de produits en croix. On trouve donc (en utilisant les coordonnées $\vz:\vt:\vzz$): \[ P_{1}=(AM)\cap(BM_{1}')\simeq\left[\begin{array}{c} x^{2}-2\,x+1+i\\ x-1\\ x^{2}-2\,x+1-i \end{array}\right]\ptv P_{2}\simeq\left[\begin{array}{c} ix^{2}-i+2\,x\\ x^{2}+1\\ -ix^{2}+i+2\,x \end{array}\right] \]

    \noindent --------- 2342848

    (1) Les ombilics, vaste sujet! Pour commencer, on a la relation formelle: \[ \left(\begin{array}{c} \vz\\ \vt\\ \vzz \end{array}\right)=\vz\,\left(\begin{array}{c} 1\\ 0\\ 0 \end{array}\right)+\vt\,\left(\begin{array}{c} 0\\ 1\\ 0 \end{array}\right)+\vzz\,\left(\begin{array}{c} 0\\ 0\\ 1 \end{array}\right) \] ce qui veut dire que les " coordonnées projectives complexes à la sauce Morley" utilisent d'une part l'origine usuelle $0:1:0$ et d'autre part deux points louches (les ombilics) comme base du système de coordonnées. Ces deux points sont louches parce qu'ils sont à l'infini ($\vt=0$) et de plus ne sont pas des points visibles ($\zeta\neq\overline{z})$.

    Cela ne les empêche pas d'apparaître en permanence. Par exemple une similitude est une collinéation laissant invariants les ombilics. Ensuite de quoi, un cercle est une conique passant par les ombilics. Vouloir se passer des ombilics revient à vouloir se passer des similitudes et des cercles. Quis tulerit Gracchos de seditione querentes !

    (2) remarque latex. Lorsque l'on gère les parenthèses, crochets, etc. soi-même, on utilise array. Lorsque l'on sous-traite tout cela, on utilise pmatrix, bmatrix, etc. Mais utiliser les parenthèses et matrix, ce n'est pas le mieux à faire.

    Cordialement, Pierre
  • Modifié (February 2022)
    pldx1 a dit :
    On pourrait essayer d'inverser la matrice $P$. Mais ce n'est pas la méthode la plus efficace. Il vaut mieux inverser les matrices $\boxed{\Gamma}$ et $\boxed{m_{jk}}$.
    Bonsoir,
    Je crois comprendre ceci : transformons $\boxed{\Gamma}\doteq\tra P\cdot\boxed{m_{jk}}\cdot P$ $~~~~~~~~~~~~~$ en $~~~~~~~~~~~$ $\boxed{\Gamma}^{-1}\doteq P^{-1}\cdot\boxed{m_{jk}}^{-1}\cdot (\tra P)^{-1}$, alors \[\boxed{m_{jk}}^{-1}\doteq P \cdot \boxed{\Gamma}^{-1}\cdot\tra P\] Quel gain en effet ! En matière d'inverses, mes calculs coûtaient deux, (au moment où je dis ça, j'oublie un peu que la transposée de l'inverse est l'inverse de la transposée. Mais ta remarque sur les déterminants $5\times5$ est parfaitement convaincante) tandis qu'avec cette belle ruse il n'y en a qu'un, celui de la matrice produit $\boxed{m_{jk}}^{-1}$ que nous allons trouver. On peut vraiment dire "un seul" calcul d'inverse, vu que l'autre $ \boxed{\Gamma}^{-1}=\left(\begin{array}{ccc} 0 & 0 & 2\\ 0 & -1 & 0\\ 2 & 0 & 0 \end{array}\right) $ est fort agréable, et surtout constant si l'on s'intéresse à plusieurs exemples !
    Faisons-nous plaisir avec l'exemple 4.
    \[\boxed{m_{jk}}^{-1}\doteq P \cdot \boxed{\Gamma}^{-1}\cdot\tra P\doteq\left[\begin{array}{ccc} -2 & -3 & 8\\ 1 & -6 & 2\\ 1 & -3 & 5 \end{array}\right] \left(\begin{array}{ccc} 0 & 0 & 2\\ 0 & -1 & 0\\ 2 & 0 & 0 \end{array}\right) \left[\begin{array}{ccc} -2 & 1 &1\\ -3 & -6 & -3\\ 8 & 2 & 5 \end{array}\right]=\left[\begin{array}{ccc} 73 & 10 &13\\ 10 & 28 & 4\\ 13 & 4 & -11 \end{array}\right]\]  donne  $ \boxed{m_{jk}}\simeq\left[\begin{array}{ccc} 2 & -1 & 2\\ -1 & 6 & 1\\2 & 1 & -12 \end{array}\right] $, comme de juste.

    pldx1 a dit :
    Autrement dit, il vaut mieux passer aux matrices décrivant les coniques tangentielles.
    Argh, de nouveau larghé (sic)... En tout cas merci, Pierre.
    Est-ce que cet Algorithme locusconi ("lieu de la conique" ?) est ta création pour la circonstance, ou un objet Maple ?
    Si le commentaire sur les coniques tangentielles m'arrache un hoquet, c'est de plaisir, à l'idée que je vais bien finir par comprendre plus d'aspects géométriques de la question.
    À ce propos, peut-on voir dans $ \boxed{\Gamma}=\left(\begin{array}{ccc} 0 & 0 & 1\\ 0 & -2 & 0\\ 1 & 0 & 0 \end{array}\right) $ une parabole (à cause de $y^2=x$) ? Si oui, que vient-elle faire ici ? Et son inverse avec $y^2=4x$ ? Hum. (J'ai bien lu la phrase qui disait qu'il n'y avait pas d'hyperbole dans l'expression de $s'$ en fonction de $s$.)

    Au moment où la réponse de pappus est arrivée, c'est peut-être moi qui avais changé la première question "Comment trouver une équation de la conique ?" en une autre "Comment la trouver avec des matrices ?" donc pour le taquiner, je dirais que "pour une fois, il répondait exactement à une question posée" même si, effectivement, il lui arrive de modifier les questions ... à notre grande joie !
    En l'occurrence, c'est quand même incroyable que je n'aie pas pensé tout seul à éliminer le paramètre $s$.
    Quatre lignes de concession aux regrets
    "De mal penser la faiblesse
    De n'avoir pas fait d'études
    Les chansons de ma jeunesse
    Et de Robert Zimmermann, l'altitude"
    et hop, je redémarre ragaillardi !
    pldx1 a dit :
    remarque latex. Lorsque l'on gère les parenthèses, crochets, etc. soi-même, on utilise array. Lorsque l'on sous-traite tout cela, on utilise pmatrix, bmatrix, etc. Mais utiliser les parenthèses et matrix, ce n'est pas le mieux à faire.
    Merci pour la remarque, je vais essayer d'améliorer. Mais j'avoue : je ne comprends ni "on gère (...) on utilise array" ni "Lorsqu'on sous-traite tout cela". Peut-être "on écrit des programmes" et "on écrit en latex" ? Ou "on écrit sur le forum" ?
    Notations algébriques : quelles matrices méritent selon toi des crochets ou des parenthèses, ainsi que des $\doteq$, ou des $\simeq$ ? Pourquoi encadrer certains noms ?
    Encore un endroit où je compliquais inutilement : dans les cinq lignes de la matrice $5\times6$. Merci d'y avoir remis un ordre, au fond, très naturel !
    Amicalement,
    Swingmustard
  • Bonjour,

    1.  A propos de LaTeX. Ce § est juste une remarque de style. Ne rien y voir de normatif !  C'est simplement pour te laisser le choix d'en tenir compte... ou non. Tu utilises (1). J'utilise (2).
      (1) \left( \begin {matrix}-2 & -3 & 8\\ 1 & -6 & 2\\ 1 & -3 & 5 \end{matrix} \right)
      (2) \left( \begin{array}{ccc} -2 & -3 & 8\\ 1 & -6 & 2\\ 1 & -3 & 5 \end{array}\right)
      (3) \begin{pmatrix}-2 & -3 & 8\\ 1 & -6 & 2\\ 1 & -3 & 5 \end{pmatrix}} 
      Peut-être que (3) serait mieux pour ce que tu me sembles vouloir faire: si tu ne te sers pas des fonctionnalités de " array" , autant utiliser les diverses " xmatrix" .
    2. A propos de normalisation. J'utilise " pmatrix" pour représenter les Vector[column] --- qui représentent des points. J'utilise des " listes à la main" et des crochets pour représenter les Vector[row] --- qui représentent les droites (je veux voir les virgules qui séparent les champs). J'utilise " bmatrix" pour les Matrix qui représentent... tout un tas de choses variées. C'est juste comme cela, c'est ainsi que j'ai programmé les exportations Maple -> LyX. Les fois où je me mêle de descendre dans les sous-sols et d'écrire directement en LaTeX... je fais de mon mieux pour être conforme à ces règles.
    3. A propos de la mise en boîte des matrices. On a une conique. On lui donne un nom, soit $\mathcal{C}$. On sort de sa poche le théorème fondamental qui fonde la théorie des coniques. Ce théorème est:
      Théorème fondamental: la hessienne d'une fonction polynomiale homogène du second degré est égale à la fonction initiale. 
      Et voila, c'est fini: nous disposons de tout ce qu'il y a à savoir sur les coniques. On décide de représenter notre conique par une matrice. Quelle nom donner à cette matrice? On pourrait l'appeler MP, comme dans " moulin à poireaux" . La seule chose que l'on ne pourrait pas, serait d'utiliser le même nom que pour la conique elle-même. J'ai choisi $\boxed{\mathcal{C}}$: un carré pour une matrice carrée. On croirait du Shakespeare. Le mode d'emploi de $\boxed{\mathcal{C}}$ est  \[ P\in\mathcal{C}\Longleftrightarrow\tra{P\cdot\boxed{\mathcal{C}}\cdot P=0} \]
    4. Tout le monde connaît la formule d'Euler. Elle montre que la tangente à la courbe algébrique déterminée par un polynôme homogène se décrit par la ligne des dérivées partielles (cela donne la 'vraie' tangente, pas seulement sa direction). Pour une conique, cela se traduit par: \[ \Delta_{P}\doteq\mathrm{tangente}\left(P\right)\simeq\tra{P\cdot\boxed{\mathcal{C}}} \] Au passage, \doteq veut dire " égal par définition" , tandis que \simeq veut dire " égal à un multiplicateur près" .
    5. Passons au corollaire fondamental du théorème fondamental. On a: \[ \Delta_{P}\cdot\left(\boxed{\mathcal{C}}\right)^{-1}\cdot\tra{\Delta_{P}}=\left(\tra{P\cdot\boxed{\mathcal{C}}}\right)\cdot\left(\boxed{\mathcal{C}}\right)^{-1}\cdot\left(\tra{\boxed{\mathcal{C}}}\cdot P\right)=0 \] Autrement dit: $\tra{P\cdot\boxed{\mathcal{C}}\cdot P=0}$ est la condition pour qu'un point soit sur la conique. Elle s'appelle donc équation ponctuelle de la conique. Et par ailleurs, $\Delta_{P}\cdot\boxed{\mathcal{C}^{*}}\cdot\tra{\Delta_{P}=0}$ est la condition pour qu'une droite soit tangente à la conique. Elle s'appelle donc équation tangentielle de la conique. Au passage, nous avons remplacé la " matrice inverse" par la " matrice adjointe" : puisque tout se passe à un multiplicateur près, il ne sert à rien de traîner les dénominateurs. Au passage, n'hésitons pas à moquer, encore et encore, cette manie franco-française de faire exprès de placer les cofacteurs à la mauvaise place... pour finalement se décider à transposer le tout.
    6. Exercice. On s'intéresse à la courbe de Descartes, dont l'équation ponctuelle est: $x^{3}+y^{3}-6xy=0$. Déterminer l'équation tangentielle associée.


    Cordialement, Pierre
  • Modifié (March 2022)
    pldx1 a dit :
    Il est temps de mentionner qu'une collineation se décrit dans geogebra par:
    mH4 = {{14, 24, -16}, {-8, 12, 4}, {-4, -3, 20}}
    D' = ApplyMatrix[mH4, D]
    Bonjour !
    1) J'essaie d'utiliser ce qui précède pour tracer une conique de matrice C.
    Méthode actuelle : j'écris C(1,1)$x^2+2$C(1,2)$xy+2$C(1,3)$x+$C(2,2)$y^2$+...
    Il y a certainement moins lourd, n'est-ce pas ?

    2) J'imagine que j'ai rencontré la Formule d'Euler en cours, mais je n'avais retenu ni son nom ni son emploi.
    Voici la version (trop) particulière que je connaissais.
    Pour un point $A(a,b)$ et pour le cercle d'équation de centre $\Omega(\alpha,\beta)$, l'équation de la polaire $p_A$ peut s'écrire $$(a-\alpha)(x-\alpha)+(b-\beta)(y-\beta)=r^2$$ Cette fichue version me faisait tenter confusément, pour une conique, de retrouver une forme canonique que j'aurais tenté de "dédoubler". Autant avouer, et j'espère ainsi rassurer les lecteurs qui partagent mes ignorances : je n'arrivais pas à écrire une équation de tangente à une conique, même si existait l'intuition ou le souvenir que ce n'était pas sorcier.
    (Je ne doute pas, d'ailleurs, que la version "sans matrice" existe, mais je n'ai pas envie de la chercher pour le moment.)
    Faisons le lien avec la formule. Trouvons la matrice du cercle en partant de $(x-\alpha)^2+(y-\beta)^2)=r^2\Longleftrightarrow x^2-2\alpha x + \alpha^2+y^2-2\beta y+\beta^2-r^2=0$.
    Multiplions, comme conseillé, la "transposée du point $A$" par la matrice $3\times3$ du cercle. $$\begin{pmatrix} a &b & 1 \end{pmatrix}\begin{pmatrix} 1 &0 & -\alpha\\0 &1 & -\beta\\-\alpha & -\beta &\alpha^2+\beta^2-r^2 \end{pmatrix}=\begin{pmatrix} a- \alpha&b-\beta & -a\alpha-b\beta+\alpha^2+\beta^2-r^2 \end{pmatrix}$$ Le deuxième membre donne bien l'équation de la polaire. $$p_A:\begin{pmatrix} a- \alpha&b-\beta & -a\alpha-b\beta+\alpha^2+\beta^2-r^2 \end{pmatrix}\begin{pmatrix} x\\y\\1 \end{pmatrix}=0 \Longleftrightarrow (a-\alpha)(x-\alpha)+(b-\beta)(y-\beta)=r^2$$ [EDIT Tentative de meilleures notations suite à message de pldx1 plus loin $$\,^t\begin{pmatrix} a\\b \\1 \end{pmatrix}\cdot\begin{pmatrix} 1 &0 & -\alpha\\0 &1 & -\beta\\-\alpha & -\beta &\alpha^2+\beta^2-r^2 \end{pmatrix}=\left[\begin{array}{l} a- \alpha&b-\beta & -a\alpha-b\beta+\alpha^2+\beta^2-r^2 \end{array}\right]$$ $$p_A:\left[\begin{array}{c} a- \alpha&b-\beta & -a\alpha-b\beta+\alpha^2+\beta^2-r^2 \end{array}\right]\cdot\begin{pmatrix} x\\y\\1 \end{pmatrix}=0 \Longleftrightarrow (a-\alpha)(x-\alpha)+(b-\beta)(y-\beta)=r^2$$ Fin de EDIT]
    Désormais je n'oublierai pas, c'est merveilleusement simple, merci Pierre !
    Exemple. Pour $P$ ci-dessous, donner une équation de la polaire $p$ par rapport à la conique associée à la matrice $\boxed{C}$.
    Même question pour le point $Q$ et sa polaire $q$.

    $$\begin{pmatrix} -11&-2 &1 \end{pmatrix}\begin{pmatrix} 2 & -1 & 2\\ -1 & 6 & 1\\2 & 1 & -12 \end{pmatrix}=\begin{pmatrix} -18&0 &-36 \end{pmatrix}\simeq \begin{pmatrix} 1&0 &2 \end{pmatrix} $$ $$\begin{pmatrix} 7&-2 &1 \end{pmatrix}\begin{pmatrix} 2 & -1 & 2\\ -1 & 6 & 1\\2 & 1 & -12 \end{pmatrix}=\begin{pmatrix} 18&-18 &0 \end{pmatrix}\simeq \begin{pmatrix} 1&-1 &0 \end{pmatrix} $$ sont bien les matrices associées à $p:x=-2$ et $q:y=x$.
    (Normalement, je me serais peut-être inquiété de ce zéro dans la troisième colonne de $q$, mais bon : ça fonctionne !)

    3) Le folium de Descartes.
    Appétissant ! Généraliser ce qui précède au degré 3 m'a l'air moins simple qu'on pourrait espérer. Je n'abandonne pas pour autant.

    4) Latex : au lieu que la matrice ligne se trouve au niveau de la deuxième ligne de la matrice $3\times3$, je rêve de la lire au niveau de la troisième ligne. Serait-ce une écriture correcte ? Si oui, comment faire ?
    Amicalement,
    Swingmustard
  • Bonjour,

    1. Les matrices sous geogebra. Ce logiciel "ne semble pas être le meilleur" pour gérer le calcul matriciel. C'est ainsi que le produit "matrice ligne" par "matrice carrée" ... se passe mal (version alternative: pldx1 n'est pas fichu de se faire obéir par son logiciel). Il n'en reste pas moins que ApplyMatrix est extrêmement efficace.
    2. Note à Benêts: la polarité par rapport à une conique n'est pas la même chose que la polarité par rapport à un triangle. En effet, une conique est une courbe du second degré, tandis qu'un trigone est une courbe du troisième degré. Il est assez souvent utile de lever les ambiguïtés en utilisant les termes coni-polaire et tri-polaire.
    3. Pour ce qui est des polaires réciproques... la propriété la plus remarquable est la réciprocité polaire: si $A$ est sur la coni-polaire de $B$, alors $B$ est sur la coni-polaire de $A$. Ecrit matriciellement, cela devient "fortement évident" . Cela n'en fait pas moins une propriété fortement utile.
    4.  Et donc  \begin{eqnarray*} \tra{\left(\begin{array}{c} -11\\ -2\\ 1 \end{array}\right)}\cdot\begin{pmatrix}2 & -1 & 2\\ -1 & 6 & 1\\ 2 & 1 & -12 \end{pmatrix} & = & \left[-18,0,-36\right]\simeq\left[1,0,2\right]\\ \tra{\left(\begin{array}{c} 7\\ -2\\ 1 \end{array}\right)}\cdot\begin{pmatrix}2 & -1 & 2\\ -1 & 6 & 1\\ 2 & 1 & -12 \end{pmatrix} & = & \left[18,-18,0\right]\simeq\left[1,-1,0\right] \end{eqnarray*} gagne à être complété par  \[ \left[1,0,2\right]\wedge\left[1,-1,0\right]\simeq\left(\begin{array}{c} -2\\ -2\\ 1 \end{array}\right)\ptv\left(\begin{array}{c} -11\\ -2\\ 1 \end{array}\right)\wedge\left(\begin{array}{c} 7\\ -2\\ 1 \end{array}\right)\simeq\left[0,18,36\right] \]
    5.  Un deuxième pour être sûr. On garde la même conique et on prend $P\simeq-11:-2:1$ et $D\simeq-5:4:1$. On calcule $P\wedge D\simeq\left[1,-1,9\right]$. On détermine le pôle $E$ de la droite $PD$. Puis on détermine les polaires des points $P,D$. Et alors...
      (1) "something is happening, but what ?"
      (2) "elaborate further using" $G=-14:-5:1$ ! 



    Cordialement, Pierre.
  • Modifié (March 2022)
    Bonjour !
    2. En effet, la Géométrie classique et analytique de Jean-Denis Eiden m'avait appris à ne pas confondre ces deux polarités. Mais je ne m'étais pas posé la question du degré d'un trigone, et j'aurais beaucoup hésité à proposer "trois". Merci ! (Hésitation incompréhensible : une droite = degré 1, deux droites = degré 2 en faisant le produit des premiers membres de leurs équations cartésiennes.
    Donc trois droites... Mon problème venait sûrement de la confusion entre triangle et trigone = réunion des trois droites.)
    Amusant : la page https://fr.wikipedia.org/wiki/Droite_centrale contient l'expression polaire trilinéaire (la tri-polaire de Pierre) avec un lien très ballot puisqu'il nous dirige vers "pôle et polaire" qui ne cause que coni-polaire !
    Il faut donc aller sur la page anglaise https://en.wikipedia.org/wiki/Trilinear_polarity pour apprendre que le concept est dû à ... Jean-Victor Poncelet, tu m'étonnes.
    4. "Gagne à être complété" : j'interprète ainsi : le produit vectoriel (wedge ?) de deux droites donne leur point d'intersection. Le produit vectoriel de deux points donne la droite qui les joint. (En l'occurrence, la droite d'équation $y=-2$.)
    Merci aussi pour cette précision. Je me demande comment tu devines qu'elle m'échappait, tout en étant dans l'air depuis un moment !
    5. Pôle $E$ de la droite $P\wedge D\simeq\left[1,-1,9\right]$ ? J'utilise (3) en écrivant ceci, confirmé par ton dessin. $$\begin{pmatrix}2 & -1 & 2\\ -1 & 6 & 1\\ 2 & 1 & -12 \end{pmatrix}^{-1}\cdot\begin{pmatrix}1\\ -1\\9\end{pmatrix} \simeq\begin{pmatrix}73 & 10 &13\\ 10 &28 & 4\\13 &4 & -11 \end{pmatrix}\cdot\begin{pmatrix}1\\ -1\\9\end{pmatrix} \simeq \begin{pmatrix}-10\\ -1\\5\end{pmatrix} $$
    [EDIT Tentative de meilleure notation, suite au message de Pierre du 1er mars.
    $$\begin{pmatrix}2 & -1 & 2\\ -1 & 6 & 1\\ 2 & 1 & -12 \end{pmatrix}^{-1}\cdot\tra{\left[\begin{array}{c} 1&-1&9 \end{array}\right]} \simeq\begin{pmatrix}73 & 10 &13\\ 10 &28 & 4\\13 &4 & -11 \end{pmatrix}\cdot\tra{\left[\begin{array}{c} 1&-1&9 \end{array}\right]} \simeq \begin{pmatrix}-10\\ -1\\5\end{pmatrix} $$ Fin de EDIT]
    Polaire de $P$ connue : $\left[1,0,2\right] $. Polaire de $D$ :
    \begin{eqnarray*}\tra{\left(\begin{array}{c} -5\\4\\ 1 \end{array}\right)}\cdot\begin{pmatrix}2 & -1 & 2\\ -1 & 6 & 1\\ 2 & 1 & -12 \end{pmatrix} & = & \left[-12,30,-18\right]\simeq\left[2,-5,3\right] \end{eqnarray*} Restent les questions
    (1) "something is happening, but what ?"
    (2) "elaborate further using" $G=-14:-5:1$ !
    qui me font penser "À force, on doit disposer d'un ou plusieurs triangles autopolaires ?" mais c'est l'heure du dejeûner !
    Amicalement,
    Swingmustard
  • Modifié (March 2022)
    Suite de la réponse.
    À vous de jouer ! Il m'a fallu deux "produits scalaires", et un nuage (comme dans une tasse de lait : très léger, le nuage) de raisonnement. Peut-être ferez-vous plus rapide !
    1) Par construction de $E$, sa polaire est la droite $PD$. Or $G$ appartient à cette droite : \begin{eqnarray*}\tra{\left(\begin{array}{c} 1\\-1\\9 \end{array}\right)}\cdot\begin{pmatrix}-14 \\ -5\\1 \end{pmatrix} & =0 \end{eqnarray*} Donc la polaire de $E$ est la droite $DG$.
    2) Par définition de $E$, la polaire de $D$ passe par $E$. Passe-t-elle aussi par $G$ ? \begin{eqnarray*}\tra{\left(\begin{array}{c} 2\\-5\\3 \end{array}\right)}\cdot\begin{pmatrix}-14 \\ -5\\1 \end{pmatrix} & =0 \end{eqnarray*} Oui, ainsi la polaire de $D$ est la droite $EG$.
    [EDIT Tentative de meilleure notation, suite au message de Pierre du 1er mars.
    $$\left[\begin{array}{c} 1&-1&9 \end{array}\right]\cdot\begin{pmatrix}-14 \\ -5\\1\end{pmatrix}=0$$ $$\left[\begin{array}{c} 2&-5&3\end{array}\right]\cdot\begin{pmatrix}-14 \\ -5\\1\end{pmatrix}=0$$ Fin de EDIT]
    3) En tant qu'intersection des polaires $DG$ et $EG$, $G$ est pôle de la droite des pôles $E$ et $D$, ou encore la polaire de $G$ est la droite $DE$.
    Conclusion : merci pour ce triangle $DEG$ auto-polaire. Pardon : auto-coni-polaire, Pierre !
    Amicalement,
    Swingmustard
    P.S. Je pige enfin le (1) "Something"... Dans 5., j'ai recouru (suivant ta demande fourbesque pédagogique) à une inversion inutile de matrice pour calculer le pôle $E$ de la droite $PD$ : il est l'intersection des polaires de $P$ et de $D$, donc peut être obtenu par $\left[1,0,2\right]\wedge\left[2,-5,3\right]\simeq\begin{pmatrix}-10\\ -1\\5\end{pmatrix}$ !
    Je soupçonne que c'est pour cela qu'il t'arrive de faire des wedge de wedge de wedge qui m'échappaient pas mal. J'y retourne !

  • Modifié (March 2022)
    Bonjour,

    \cdot versus \cdot. Faire des mathématiques, cela veut dire (1) trouver des résultats, (2) trouver des preuves, (3) vérifier que les preuves prouvent ce qu'elles sont sensées prouver. Dans ce contexte, le choix des notations est une question centrale. De bonnes notations, ce sont des notations qui facilitent l'apparition des résultats, puis l'apparition des preuves. Sans support pour l'intuition, il y a moins d'intuition!

    De ce point de vue, utiliser un \cdot ($\cdot$) pour dire une chose et, ensuite, utiliser le même \cdot pour dire totalement autre chose est une erreur. Le premier \cdot sert à décrire un calcul matriciel. Autrement dit, Alice mesure à sa façon des trucs et des machins et, ensuite, fait des calculs avec ce qu'elle a mesuré. Le modèle est $\left[a,b\right]\cdot\left(\begin{array}{c} c\\ d \end{array}\right)=ac+bd$. De son côté, Bob mesure à sa façon les trucs et les machins et, ensuite, fait des calculs avec ce qu'il a mesuré. Le modèle est $\left[a',b'\right]\cdot\left(\begin{array}{c} c'\\ d' \end{array}\right)=a'c'+b'd'$.

    Le deuxième \cdot sert, à tort, à désigner un produit scalaire, i.e. sert à noter $\left\langle A|B\right\rangle $ sous la forme $\overrightarrow{A}\cdot\overrightarrow{B}$. Si l'on a $\overrightarrow{A}=\begin{pmatrix}a\\ b \end{pmatrix}$, $\overrightarrow{B}=\begin{pmatrix}c\\ d \end{pmatrix}$ et si l'on a choisi le prodscal canonique, cela donne $\left\langle A|B\right\rangle =ac+bd$. C'est le moment de ne pas dire: puisque cela vaut pareil, c'est que c'est pareil. Cela n'est pas le cas du tout!

    Le \cdot matriciel prend en entrée une ligne et une colonne, autrement dit deux objets ayant des natures différentes, c'est à dire vivant dans deux copies différentes de l'espace abstrait $\mathbb{C}^{n}$. Par exemple la ligne peut représenter une droite, et la colonne peut représenter un point. Et alors le \cdot des deux nous dit s'ils incident tous les deux, ou s'ils n'incident pas. Ce \cdot est un opérateur universel, c'est le même pour Alice et pour Bob.

    Faire un changement de repère, cela veut dire remplacer $\left[a,b\right]$ par $\left[a',b'\right]$ et $\left(\begin{array}{c} c\\ d \end{array}\right)$ par $\left(\begin{array}{c} c'\\ d' \end{array}\right)$. Supposons que l'on utilise le changement de repère décrit par $\left(\begin{array}{c} c'\\ d' \end{array}\right)=\boxed{M}\cdot\left(\begin{array}{c} c\\ d \end{array}\right)$, autrement dit une transformation linéaire agissant sur les colonnes. Supposons en outre que l'on souhaite que le \cdot reste invariant, imposant que \[ \forall c,\forall d:\:\left[a,b\right]\cdot\left(\begin{array}{c} c\\ d \end{array}\right)=\left[a',b'\right]\cdot\left(\begin{array}{c} c'\\ d' \end{array}\right)=\left[a',b'\right]\cdot\boxed{M}\cdot\left(\begin{array}{c} c\\ d \end{array}\right) \] et donc que $\left[a,b\right]=\left[a',b'\right]\cdot\boxed{M}$. La matrice $\boxed{M}$ (agissant à droite) décrit donc la transformation réciproque agissant sur les lignes. Or la transformation $\boxed{M}$ est inversible, sinon il y aurait des colonnes qui ne seraient pas décrites dans le nouveau système. On a donc $\left[a',b'\right]=\left[a,b\right]\cdot\boxed{M}^{-1}$. Et alors l'intrinséquitude en devient évidente, selon: \[ \left[a',b'\right]\cdot\left(\begin{array}{c} c'\\ d' \end{array}\right)=\left(\left[a,b\right]\cdot\boxed{M}^{-1}\right)\cdot\left(\boxed{M}\cdot\left(\begin{array}{c} c\\ d \end{array}\right)\right)=\left[a,b\right]\cdot\left(\boxed{M}^{-1}\cdot\boxed{M}\right)\cdot\left(\begin{array}{c} c\\ d \end{array}\right)=\left[a,b\right]\cdot\left(\begin{array}{c} c\\ d \end{array}\right) \]

    Passons au produit scalaire. Comment faire pour qu'il survive à un changement de repère ? C'est une excellente question, quoique totalement mal posée. Les maths, cela ne consiste pas à choisir des axiomes au hasard et à prier un dieu ou un autre dans l'espoir qu'il en résulte quelque chose. La question est donc: sachant que le produit scalaire survit aux changements de repères, comment le décrire pour que cette propriété en devienne évidente ?

    La réponse est connue. On a  \[ \left\langle A|B\right\rangle \doteq\tra A\cdot\boxed{\mathcal{C}}\cdot B \] pour une certaine matrice $\boxed{\mathcal{C}}$ ayant un certain nombre de propriétés, en particulier d'être symétrique. L'intrinséquitude s'écrit: \[ \forall A,\forall B:\:\tra A\cdot\boxed{\mathcal{C}}\cdot B=\tra{A'}\cdot\boxed{\mathcal{C}'}\cdot B'=\tra{\left(\boxed{M}\cdot A\right)}\cdot\boxed{\mathcal{C}'}\cdot\left(\boxed{M}\cdot B\right) \] ce qui impose que $\boxed{\mathcal{C}}=\tra{\boxed{M}}\cdot\boxed{\mathcal{C}'}\cdot\boxed{M}$ c'est à dire que: $\boxed{\mathcal{C}'}=\tra{\boxed{M}^{-1}}\cdot\boxed{\mathcal{C}}\cdot\boxed{M}^{-1}$. Toute ressemblance avec les coniques n'est ni fortuite ni volontaire, mais simplement inévitable.

    Tout ce qui précède n'est autre que le principe de relativité: les équations ont la même forme dans tous les repères. Mais on peut aussi décider de sortir Platon de sa naphtaline. Si Alice, de sa caverne, voit $A,B,\left\langle A|B\right\rangle $ et que Bob, de sa caverne, voit $A',B',\left\langle A'|B'\right\rangle $ et que $\left\langle A|B\right\rangle =\left\langle A'|B'\right\rangle $, c'est qu'il existe dans le vrai monde extérieur des objets tintrinsèques $\overrightarrow{A}$ et $\overrightarrow{B}$ tels que les $\left\langle A_{j}|B_{j}\right\rangle $ soient l'ombre portée par $\left\langle \overrightarrow{A}|\overrightarrow{B}\right\rangle $, le prodscal tintrinsèque des objets tintrinsèques. Vous reprendrez bien un peu de viande grillée (dans une caverne ou une autre) ?

    Cordialement, Pierre.
  • Modifié (March 2022)
    Bonjour !
    1. @pldx1 Heureusement que j'avais mis des guillemets à "produit scalaire".
    Heureusement que j'ai écrit en début de discussion que je ne prétends pas démontrer quoi que ce soit.
    Mais merci quand même pour la grillade  :) J'ai tenté de rectifier mes notations sur les trois derniers messages.
    L'asymétrie de nos connaissances mathématiques fait que je n'ai qu'une chose à te proposer de corriger : ta conjugaison du verbe "résoudre"  ;)

    2. Où j'ai cru un peu vite que "conique bitangente entraîne stable par toute collinéation ayant pour points fixes les ceusses qu'il faut".
    a) Imaginons. Vous souhaitez fabriquer une des collinéations les plus courantes : celles à trois points fixes, dixit pappus. Par exemple : $U(1,-1)$, $V(2,3)$, $W(-2,1)$. Pour changer, vous ne la définissez pas par prolongement à partir d'une conique. Vu le fonctionnement avec les matrices, il ne vous reste plus qu'à fixer trois valeurs propres arbitraires, dont $\begin{pmatrix}1\\-1\\1\end{pmatrix}$, $\begin{pmatrix}2\\3\\1\end{pmatrix}$, $\begin{pmatrix}-2\\1\\1\end{pmatrix}$ seront les vecteurs propres. En posant $P=\begin{pmatrix}1&2&-2\\-1&3&1\\1&1&1\end{pmatrix}$, vous obtenez par exemple \[ \boxed{h_{6}}\doteq P\cdot\left[\begin{array}{ccc}1&0&0\\0&2&0\\0&0&-1\end{array}\right]\cdot P^{-1}\simeq\left[\begin{array}{ccc}2&10&22\\14&21&-7\\10&1&5\end{array}\right] \] (J'aurais bien fait précéder de l'inverse de la racine cubique du déterminant, comme fait Pierre, mais on n'a pas un entier cette fois.) Vous faites dessiner ça par un certain logiciel formidable. Vous vous frottez les mains, tentant d'appliquer ce que vous croyez entraver aux faisceaux bitangents. En ce qui me concerne : que chaque paire, parmi nos trois points, peut constituer les points de contact, faisant du troisième point le pôle de la corde des deux premiers.
    En plus clair : si $U$ et $V$ sont les points de contact, $W$ sera le pôle de $UV$.
    b) On a vu que pour connaître ce faisceau (parmi les trois associés à $H_6$), on pourrait sûrement travailler sur une matrice $6\times6$.
    c) Bricolage. Mais j'ai d'abord essayé avec geogebra de trouver un représentant. Survolant des fils de 2014 ou 2017,  notamment https://les-mathematiques.net/vanilla/index.php?p=/discussion/1553040/le-faisceau-bitangent (je découvre !) j'apprends que la médiane de $UVW$ issue de $W$ passe par le centre de la conique. (Désolé de l'avoir ignoré ou oublié. Merci de ne pas taper.) Je place donc un centre $I$ courant de la future conique sur cette médiane, d'où quatre premiers points : $U,V$ et leurs symétriques par rapport à $I$. En faisant varier $I$ et un cinquième point, à qui je demande aussi d'avoir des coordonnées entières, j'obtiens la belle ellipse bleue, de matrice \[ \boxed{C}\simeq\left[\begin{array}{ccc}4&-1&-7\\-1&2&0\\-7&0&6\end{array}\right] \] C'est déjà un peu chouette.
    d) Cependant... Surprise, en croyant (à tort) que son image par la collinéation serait elle-même !
    Cette ellipse m'apprend encore une chose : être bitangente aux points de contact imposés ne suffit pas pour être stable par $H_6$ ! Quelle condition ajouteriez-vous ?
    e) On se rappelle peut-être que la matrice $6\times6$ avait, de manière non encore comprise, des valeurs propres en progression géométrique. Inspectant certaines des matrices déjà rencontrées et fabriquées en composant deux involutions, je constate qu'elles partagent cette propriété.
    Or mon choix $1,2,-1$ ne l'avait pas, cette propriété.
    L'espoir renaît... Tentons le coup avec les valeurs propres $1,-2,4$ qui donnent \[ \boxed{h_{7}}\doteq\dfrac{1}{14}\left[\begin{array}{ccc}-13&12&18\\15&5&-3\\9&3&-13\end{array}\right] \]
    Comme on voit, ça ne suffit pas non plus.
    f) État des lieux : les diverses collinéations auxquelles j'aboutis semblent laisser le faisceau $(\left\{ U,V\right\},W)$ globalement stable,
    mais je dois chercher encore ce qui caractérise celles d'entres elles qui le laissent stable conique par conique.
    Si ça se trouve, pldx1 l'a déjà dit plus haut et je n'ai pas encore compris, ou il a donné assez d'outils pour se débrouiller.
    Je dois relire ce qu'il disait le 8 février, entre autres.
    J'y revas.
    Amicalement,
    Swingmustard
  • Modifié (March 2022)
    Bonsoir,
    Dans son message #2340738,
    pldx1 a dit :
    On se donne une conique passant par $B,C$ et tangente à $AB,AC$. [...]
    3. La conique est donc invariante dans la collinéation où chaque point de $BC$ est multiplié par $+1$ et le point $A$ est multiplié par $-1$. Rem: cela est tout à fait évident sur l'équation $Kx^{2}-yz=0$.
    Visiblement, c'est moins évident pour moi, de même que la suite. Je ne risque pas de me plaindre de manquer de grain à moudre !
    Merci Pierre, prenant enfin les bonnes valeurs propres indiquées $1,1,-1$, il vient \[ \boxed{h_{8}}\doteq-\dfrac{1}{7}\left[\begin{array}{ccc}-1&2&10\\4&6&-5\\4&-1&2\end{array}\right] \] collinéation qui laisse $C$ globalement invariante.
    J'aurais bien aimé, à l'époque des études, qu'on me dise que la théorie spectrale sous-tend également d'aussi intéressantes situations.
    Mais aujourd'hui c'est chouette aussi !
    Amicalement,
    Swingmustard
    P.S. Pierre a même dit : On écrit la procédure: \begin{eqnarray*} q_{1} & := & \mathrm{Matrix}\left(\left[\Delta\wedge\mathrm{Randomrow}(3),\Delta\wedge\mathrm{Randomrow}(3),M\right]\right) \\ q_{2} & := & \mathrm{convert} \left(\mathrm{FActor}\left(q_{1}\cdot\mathrm{Diag}(1,1,-1)\cdot q_{1}^{-1}\right),\mathrm{listlist}\right) \\ M_{p} & := & \mathrm{Matrix}\circ \mathrm{unapply}(q_{2},x,y,t)\circ OP \end{eqnarray*}
    La ligne du milieu est (depuis un mois !) exactement ce que je viens (enfin) de faire, je crois. Le gars est un devin. À ma décharge, je ne connais pas ce langage, et ai été perdu à partir de "Random".
  • Bonsoir.

    Voici quelques questions qui devraient t'aider à faire le point.

    Question principale 1. On se donne les matrices  \[ M=\begin{pmatrix}2 & 10 & 22\\ 14 & 21 & -7\\ 10 & 1 & 5 \end{pmatrix}\ptv P=\begin{pmatrix}1 & 2 & -2\\ -1 & 3 & 1\\ 1 & 1 & 1 \end{pmatrix} \] Sachant que les colonnes propres de la matrice $H$ sont les colonnes de la matrice $P$, quelles sont les lignes propres de cette même matrice $H$?

    Question auxiliaire 1. On se donne une conique par sa matrice  \[ \boxed{C}=\begin{pmatrix}4 & -1 & -7\\ -1 & 2 & 0\\ -7 & 0 & 6 \end{pmatrix}, \] puis on change les coordonnées des points par  \[ X\doteq\left(\begin{array}{c} x\\ y\\ t \end{array}\right)\mapsto X'\doteq\left(\begin{array}{c} x'\\ y'\\ t' \end{array}\right)=M\cdot\left(\begin{array}{c} x\\ y\\ t \end{array}\right) \] Quelle est la nouvelle écriture matricielle de $\mathcal{C}$ (à utiliser avec les $X'$) ?

    Question auxiliaire 2. Une conique dégénérée est une conique qui se décompose en deux droites. Comment s'écrit la matrice d'une telle conique ?

    Question auxiliaire 3. On suppose que $M$ possède une base de colonnes propres (et donc aussi une base de lignes propres). Montrer que $\widehat{M}$ admet une base de colonnes propres correspondant à des coniques dégénérées. On rappelle que  \[ \widehat{M}=\dfrac{1}{196}\,\left[\begin{array}{cccccc} 4 & 196 & 100 & 56 & 280 & 40\\ 100 & 441 & 1 & 420 & 42 & 20\\ 484 & 49 & 25 & -308 & -70 & 220\\ 20 & 294 & 10 & 182 & 224 & 102\\ 220 & -147 & 5 & 392 & 98 & 72\\ 44 & -98 & 50 & 294 & 0 & 230 \end{array}\right] \] Quelles sont les valeurs propres de $M$ ?

    Question auxiliaire 4. Quelles sont les conditions pour que $M$ laisse invariante une conique non dégénérée (celle que l'on veut... ou une autre) ?

    Question principale 2. Quelles sont les conditions pour que $M$ laisse invariante la conique passant par $U,V$ et ayant $WU,WV$ comme tangentes ?

    Cordialement, Pierre.

  • Bonjour,
    pldx1 a dit: Question principale 1. On se donne les matrices  \[ M=\begin{pmatrix}2 & 10 & 22\\ 14 & 21 & -7\\ 10 & 1 & 5 \end{pmatrix}\ptv P=\begin{pmatrix}1 & 2 & -2\\ -1 & 3 & 1\\ 1 & 1 & 1 \end{pmatrix} \] Sachant que les colonnes propres de la matrice $H$ sont les colonnes de la matrice $P$, quelles sont les lignes propres de cette même matrice $H$?
    Si $P^{-1}\cdot M\cdot P= \Delta$ alors $Q\cdot M\cdot Q^{-1}= \Delta$ si l'on choisit $Q=P^{-1}$. Evidemment, cela suppose de disposer d'une famille de colonnes propres qui forment une base de l'espace des colonnes. Mais n'est-ce pas le cas ici ? Cordialement, Pierre
  • Modifié (March 2022)
    Bonjour,
    Bien d'accord. J'y vais lentement, pour les gens comme moi.
    Question principale 1 \[ M=\begin{pmatrix}2 & 10 & 22\\ 14 & 21 & -7\\ 10 & 1 & 5 \end{pmatrix}\] a été construite pour que \[ P_1=\begin{pmatrix} 1\\  -1\\1 \end{pmatrix}\ptv P_2=\begin{pmatrix} 2\\ 3\\1 \end{pmatrix}\ptv P_3=\begin{pmatrix} -2\\  1\\1 \end{pmatrix} \] soient des colonnes propres. Par exemple $P_2$ est, au départ, colonne propre pour la valeur propre 2 même si, vu le contexte, cela revient à une valeur propre 1, i.e. cette colonne propre correspond au point affine $V(2,3)$ fixe pour la collinéation : \[ M\cdot P_2=\begin{pmatrix}2 & 10 & 22\\ 14 & 21 & -7\\ 10 & 1 & 5 \end{pmatrix}\cdot\begin{pmatrix} 2\\ 3\\1 \end{pmatrix}=\begin{pmatrix} 56\\ 84\\28 \end{pmatrix}\simeq2P_2\simeq P_2\] Comme le fait remarquer pldx1, $P$ est inversible d'inverse notée $Q\doteq\begin{pmatrix}2 & -4 & 8\\ 2 & 3 & 1\\ -4 & 1 & 5 \end{pmatrix}$. Donc $$M\cdot P=P\cdot\begin{pmatrix}1 & 0 & 0\\0 & 2 & 0\\ 0 &0 & -1 \end{pmatrix}\Longleftrightarrow Q\cdot M=\begin{pmatrix}1 & 0 & 0\\0 & 2 & 0\\ 0 &0 & -1 \end{pmatrix}\cdot Q$$ Pierre nous dit à raison que $Q$ (Edit : non, $M$ !) a des lignes propres. Par exemple \[ Q_3\cdot M=\begin{bmatrix}-4&1& 5 \end{bmatrix}\cdot\begin{pmatrix}2 & 10 & 22\\ 14 & 21 & -7\\ 10 & 1 & 5 \end{pmatrix}=\begin{bmatrix}56&-14& -70 \end{bmatrix}\simeq Q_3\] Interprétation géométrique. Les colonnes propres correspondent à des points fixes $U$, $V$, $W$.
    On a vu (ailleurs) que les lignes propres décrivent respectivement les droites stables $VW$, $WU$, $UV$. Ceci nous fait quand même un peu plaisir : rappelons-nous qu'on parle d'une collinéation, donc une transformation qui envoie (par exemple) la droite $UV$ sur une droite passant par $U'=U$ et par $V'=V$, i.e. sur elle-même, ouf !
    Question auxiliaire 1 et un peu plus.
    Le même genre de considération me semble montrer que
    a) si une droite quelconque $D$ a pour matrice-ligne $L$ (associée à l'équation $L\cdot\begin{pmatrix}x\\y\\t \end{pmatrix}=0 $ de cette droite), alors l'image de $D$ a pour matrice $L.M^{-1}$.
    b) si une conique a pour matrice carrée $\boxed{C}$, son image a pour matrice $\,^t M^{-1}\cdot\boxed{C}\cdot M^{-1}$.
    Question auxiliaire 2. "Une conique dégénérée est une conique qui se décompose en deux droites. Comment s'écrit la matrice d'une telle conique ?" J'arrive à transformer l'équation $$(ax+by+ct)(dx+ey+ft)=0$$ en \[ \begin{pmatrix}x &y &t\end{pmatrix}\cdot \begin{pmatrix}a & d\\b &e\\c &f \end{pmatrix}\cdot \begin{pmatrix}d &e &f\\a & b &c \end{pmatrix}\cdot \begin{pmatrix}x\\y\\t \end{pmatrix}=0\] Je propose donc \[\boxed{C}= \begin{pmatrix}a & d\\b &e\\c &f \end{pmatrix}\cdot \begin{pmatrix}d &e &f\\a & b &c \end{pmatrix}\] mais je sens que je passe à côté de quelque chose ;-)
    Merci et à bientôt,
    Swingmustard
  • Modifié (March 2022)
    Bonjour,
    Oui, tout à fait.
    mais je sens que je passe à côté de quelque chose
    Il est possible d'avancer en utilisant la matrice que tu as trouvée. Par la suite, on pourra toujours se demander s'il y a plus "stratosphérique".
    Cordialement, Pierre.
  • Bonsoir,
    pldx1 a dit :

    Question auxiliaire 3. On suppose que $M$ possède une base de colonnes propres (et donc aussi une base de lignes propres). Montrer que $\widehat{M}$ admet une base de colonnes propres correspondant à des coniques dégénérées. On rappelle que  \[ \widehat{M}=\dfrac{1}{196}\,\left[\begin{array}{cccccc} 4 & 196 & 100 & 56 & 280 & 40\\ 100 & 441 & 1 & 420 & 42 & 20\\ 484 & 49 & 25 & -308 & -70 & 220\\ 20 & 294 & 10 & 182 & 224 & 102\\ 220 & -147 & 5 & 392 & 98 & 72\\ 44 & -98 & 50 & 294 & 0 & 230 \end{array}\right] \] Quelles sont les valeurs propres de $M$ ?
    Coquille pour "Quelles sont les valeurs propres de $\widehat{M}$ ?" ?
    Si oui, j'ai des valeurs propres $4,-2,-1,1,2,1$ en vue. Qui sont les carrés et les produits deux à deux de celles $1,2,-1$ de $M$. (J'ignore encore pourquoi, bien que ça ne surprenne pas.)
    Un vecteur propre $\begin{pmatrix}4\\9\\1\\6\\3\\2\end{pmatrix}$ auquel je trouve un air très $\begin{pmatrix}a^2\\b^2\\c^2\\ab\\bc\\ca\end{pmatrix}$, un autre $\begin{pmatrix}8\\-3\\-5\\5\\-8\\-3\end{pmatrix}$, puis $\begin{pmatrix}8\\4\\-40\\-9\\6\\11\end{pmatrix}$, et encore deux que j'ai presque, et le sixième pas du tout. Je travaille aussi l'algèbre bilinéaire, sans trop de progrès actuel.
    D'autres présentations de la conique dégénérée : \[\boxed{C}= \begin{pmatrix}a & d\\b &e\\c &f \end{pmatrix}\cdot \begin{pmatrix}d &e &f\\a & b &c \end{pmatrix}= \begin{pmatrix}a \\b\\c\end{pmatrix}\cdot \begin{pmatrix}d &e &f\end{pmatrix}+ \begin{pmatrix} d\\e\\f \end{pmatrix}\cdot \begin{pmatrix}a & b &c \end{pmatrix}= \begin{pmatrix}a & d&0\\b &e&0\\c &f &0 \end{pmatrix}\cdot \begin{pmatrix}d &e &f\\a & b &c \\0&0&0\end{pmatrix}\]
    (Hum.) Juste pour dire que je n'abandonne pas, et que j'apprécie d'avoir encore un peu de temps pour chercher.
    Amicalement,
    Swingmustard
  • Modifié (March 2022)
    Bonjour,
    pldx1 a dit: On remarquera que $\tra{\Delta_{j}}\cdot\Delta_{k}\neq\tra{\Delta_{k}}\cdot\Delta_{j}$ ne change rien à l'affaire: c'est la matrice symétrique: $\left(\tra{\Delta_{j}}\cdot\Delta_{k}+\tra{\Delta_{k}}\cdot\Delta_{j}\right)/2$ qui détermine la conique.  
    pldx1 eût mieux fait de dire: "il sera utile de remarquer". Quant au $\widehat M$ qui est sorti sans mettre son chapeau: oui, bien sûr !

    Cordialement, Pierre
  • Journée de "$M$ oder $M^{-1}$ ? Il faut choisir !"
    1) Mon erreur d'interprétation.
    pldx1 a dit le 31 janvier.
    Quelles sont les coniques qui sont invariantes sous l'action de la collineation $M_h$ ?   Cette action est $\def\tra#1{{{\vphantom{#1}}^{t}{#1}}} \def\linf{\mathcal{L}_{\infty}}$  \[ \phi:\mathcal{C}\mapsto\tra{M_{h}\cdot\mathcal{C}\cdot M_{h}} \]
    Du coup, quand
    Swingmustard dit le 8 mars
    Si une conique a pour matrice carrée $\boxed{C}$, son image a pour matrice $\,^t M^{-1}\cdot\boxed{C}\cdot M^{-1}$.
    il se demande s'il est en conflit, ne réalisant pas du tout que
    pldx1 a dit le 1er février
    L'action de la collineation $M_{h}$ sur une conique $\mathcal{C}$ peut se décrire par:  \[ \widehat{h}:\boxed{\mathcal{C}}\mapsto\tra{M_{h}^{-1}}\cdot\boxed{\mathcal{C}}\cdot M_{h}^{-1} \]
    Conclusion. Swingmustard n'est pas en conflit, ni n'a vu que la formule du 31 janvier est un cas particulier ($C$ invariante) de celle du 1er février.
    2) Mon autre erreur d'interprétation : ligne propre de $M$ ou de $M^{-1}$ ? Car malheureusement, les deux semblent coexister !
    Swingmustard a dit :
    Pierre nous dit à raison que $Q$ a des lignes propres. Par exemple \[ Q_3\cdot M=\begin{bmatrix}-4&1& 5 \end{bmatrix}\cdot\begin{pmatrix}2 & 10 & 22\\ 14 & 21 & -7\\ 10 & 1 & 5 \end{pmatrix}=\begin{bmatrix}56&-14& -70 \end{bmatrix}\simeq Q_3\]
    D'abord, je voulais bien sûr dire $M$ et pas $Q$. Plus grave : je ne me rends pas compte que ce que je propose, c'est ce que pldx1 appellerait une ligne propre de $M_{h}^{-1}$, si on confronte ce que
    pldx1 a dit le 1er février
    On sait qu'une matrice dispose de colonnes propres (décrivant des points), mais aussi de lignes propres (décrivant des droites). On a donc: $\Delta_{j}\cdot M_{h}^{-1}=\lambda_{j}^{-1}\Delta_{j}$.
    et ce que
    pldx1 dit le 3 mars
    Sachant que les colonnes propres de la matrice $H$ sont les colonnes de la matrice $P$, quelles sont les lignes propres de cette même matrice $H$?
    Ma confusion me fait passer complètement à côté de $\Delta_{j}\cdot M_{h}^{-1}=\lambda_{j}^{-1}\Delta_{j}$.
    Mais bon, réaliser ces erreurs me fait avancer, et j'entrevois de plus en plus le profit à tirer de ce que
    pldx1 a dit le 1er février
    \[ \widehat{h}\left(\tra{\Delta_{j}}\cdot\Delta_{k}\right)=\tra{M_{h}^{-1}}\cdot\tra{\Delta_{j}}\cdot\Delta_{k}\cdot M_{h}^{-1}=\left(\lambda_{j}^{-1}\lambda_{k}^{-1}\right)\;\left(\tra{\Delta_{j}}\cdot\Delta_{k}\right) \]
    J'y reconnais enfin la matrice $(3\times1)\times(1\times3)\simeq3\times3$ actuellement à l'honneur, et d'autres blocages sont en train d'être levés.
    Patience, patience !
    Amicalement,
    Swingmustard
  • Bonsoir, 

     Si l'on suppose $M$ inversible, une colonne propre de $M$ est aussi une colonne propre de $M^{-1}$ (évidemment, la valeur propre associée est remplacée par son inverse). Et, de même, une ligne propre de $M$ est aussi une ligne propre de $M^{-1}$.
     
    Et c'est pour cela que le chat est retombé sur ses pattes lorsque le pldx1 du 31 janvier a dit que: l'action de la collinéation $M_{h}$ sur la conique est $\phi:\mathcal{C}\mapsto\tra{M_{h}\cdot\mathcal{C}\cdot M_{h}}$. Mais cela reste faux! D'ailleurs, le pldx1 du 1er février a dit: l'action de la collineation $M_{h}$ sur une conique $\mathcal{C}$ peut se décrire par: $\widehat{h}:\boxed{\mathcal{C}}\mapsto\tra{M_{h}^{-1}}\cdot\boxed{\mathcal{C}}\cdot M_{h}^{-1}$. Et cette affirmation-là, c'est la bonne !

    Et, vu que le lecteur a rectifié de lui-même, "bravo, le lecteur" ! 

    Cordialement, Pierre.
  • Modifié (March 2022)
    Bonjour !
    LA FABRIQUE DE LA LIGNE PROPRE.
    Soit la matrice inversible $M=\begin{pmatrix}8&-2\\15&-3\end{pmatrix}$.
    Soit $P=\begin{pmatrix}1&2\\3&5\end{pmatrix}$ une matrice de colonnes propres.
    Soit $D=\begin{pmatrix}2&0\\0&3\end{pmatrix}$ une matrice diagonale de valeurs propres de $M$.
    $$MP=PD$$ $$P^{-1}M^{-1}=D^{-1}P^{-1}$$Ceci fait dire à pldx1 que $M$ (j'aurais peut-être dit $M^{-1}$, mais je lui fais confiance) a des lignes propres.
    J'ajoute que ce qui précède montre qu'on peut prendre les lignes de $P^{-1}$. Comme l'a déjà dit pldx1, elles seront propres pour les valeurs propres communes à $M^{-1}$ et à $D^{-1}$, qu'on sait être les inverses de celles de $M$. Vérification : $$P^{-1}M^{-1}=\begin{pmatrix}-5&2\\3&-1\end{pmatrix}\times\dfrac{1}{6}\begin{pmatrix}-3&2\\-15&8\end{pmatrix}=\dfrac{1}{6}\begin{pmatrix}-15&6\\6&-2\end{pmatrix}=\begin{pmatrix}1/2&0\\0&1/3\end{pmatrix}\begin{pmatrix}-5&2\\3&-1\end{pmatrix}=D^{-1}P^{-1}$$ Soit encore, en insistant sur les lignes propres trouvées $$\begin{pmatrix}-5&2\end{pmatrix}M^{-1}=\dfrac{1}{2}\begin{pmatrix}-5&2\end{pmatrix}\rm{~~~~~~~~~;~~~~~~~~~~~}\begin{pmatrix}3&-1\end{pmatrix}M^{-1}=\dfrac{1}{3}\begin{pmatrix}3&-1\end{pmatrix}$$ Dès le 1er février (!), pldx1 le notait ainsi : $\Delta_{j}\cdot M_{h}^{-1}=\lambda_{j}^{-1}\Delta_{j}$. Il ajoute \[ \widehat{h}\left(\,^t{\Delta_{j}}\cdot\Delta_{k}\right)=\,^t{M_{h}^{-1}}\cdot\,^t{\Delta_{j}}\cdot\Delta_{k}\cdot M_{h}^{-1}=\left(\lambda_{j}^{-1}\lambda_{k}^{-1}\right)\;\left(\,^t{\Delta_{j}}\cdot\Delta_{k}\right) \]"Cela nous donne une base de coniques propres.
    La relation $\lambda_{1}\lambda_{3}=\lambda_{2}^{2}$ fait qu'il existe un sous-espace propre de dimension $2$, engendré par $AB\times AC$ et par $BC^{2}$: c'est le faisceau bitangent dont nous n'arrêtons pas de causer.
    On remarquera que $\,^t{\Delta_{j}}\cdot\Delta_{k}\neq\,^t{\Delta_{k}}\cdot\Delta_{j}$ ne change rien à l'affaire: c'est la matrice symétrique: $\left(\,^t{\Delta_{j}}\cdot\Delta_{k}+\,^t{\Delta_{k}}\cdot\Delta_{j}\right)/2$ qui détermine la conique."
    Actuellement, je travaille sur $$M=\begin{pmatrix}2&10&22\\14&21&-7\\10&1&5\end{pmatrix}\rm{~~~~~~~~~;~~~~~~~~~~~}M^{-1}\simeq\begin{pmatrix}8&-2&-38\\-10&-15&23\\-14&7&-7\end{pmatrix}$$
    J'en suis à $$P^{-1}\simeq\begin{pmatrix}4&-1&-5\\2&3&1\\1&-2&4\end{pmatrix}$$ Ensuite j'espère que combiner ces lignes correctement me donnera, d'une manière nouvelle pour moi qui, jusqu'à présent, bricolais pour les obtenir, des vecteurs propres de $\widehat h$ !
    Amicalement,
    Swingmustard
  • Bonsoir,
    En appelant $Q_i$ la $i$ème ligne de $P^{-1}$, j'obtiens comme de juste trois droites doubles passant par $U$, $V$, $W$ $$\,^t Q_1\cdot Q_1\simeq\begin{pmatrix}16&-4&-20\\-4&1&5\\-20&5&25\end{pmatrix}~~;~~\,^t Q_2\cdot Q_2\simeq\begin{pmatrix}4&6&2\\6&9&3\\2&3&1\end{pmatrix}~~; ~~\,^t Q_3\cdot Q_3\simeq\begin{pmatrix}1&-2&4\\-2&4&-8\\4&-8&16\end{pmatrix}$$ et trois paires de droites passant par les mêmes points $U$, $V$, $W$ $$\,^t Q_1\cdot Q_2+\,^t Q_2\cdot Q_1\simeq\begin{pmatrix}8&5&-3\\5&-3&-8\\-3&-8&-5\end{pmatrix}~~; ~~\,^t Q_2\cdot Q_3+\,^t Q_3\cdot Q_2\simeq\begin{pmatrix}4&-1&9\\-1&-12&10\\9&10&8\end{pmatrix}$$ $$\,^t Q_1\cdot Q_3+\,^t Q_3\cdot Q_1\simeq\begin{pmatrix}8&-9&11\\-9&4&6\\11&6&-40\end{pmatrix}$$ Il y a certainement déjà tout dans les messages précédents de pldx1 (notamment 8 février, 14 et 15, 3 mars), mais je continue de comprendre lentement.
    pldx1 a dit le 14 février

    (3) La base " selon la théorie" de $\ker\left(\mathcal{H}_{4}-1\right)$ est $1:9/2:2:0:-3:0\ptv0:1:4:0:-2:0$. Pour faire joli, on peut prendre " notre conique" et un cercle. Les " vecteurs" proposés conviennent... mais ne vont pas faciliter le calcul numérique: le 2eme " vecteur" proposé ne contient qu'une dose de la deuxième conique pour 16 doses de la première (alors qu'on préférerait le contraire).
    Ceci entre certainement en jeu pour "doser des coniques" dégénérées et fabriquer des non dégénérées. À suivre.
    Amicalement,
    Swingmustard

  • Bonjour,
    Où en sommes-nous du programme aimablement proposé par pldx1 le 3 mars ?
    Question principale 1. On se donne les matrices  \[ M=\begin{pmatrix}2 & 10 & 22\\ 14 & 21 & -7\\ 10 & 1 & 5 \end{pmatrix}\ptv P=\begin{pmatrix}1 & 2 & -2\\ -1 & 3 & 1\\ 1 & 1 & 1 \end{pmatrix} \] Sachant que les colonnes propres de la matrice $H$ sont les colonnes de la matrice $P$, quelles sont les lignes propres de cette même matrice $H$?
    Les lignes propres de $H$ sont les lignes de la matrice $P^{-1}\simeq\begin{pmatrix}2 & -4 & 8\\ 2 & 3 & 1\\ -4 & 1 & 5 \end{pmatrix}$.
    Question auxiliaire 1. On se donne une conique par sa matrice  \[ \boxed{C}=\begin{pmatrix}4 & -1 & -7\\ -1 & 2 & 0\\ -7 & 0 & 6 \end{pmatrix}, \] puis on change les coordonnées des points par  \[ X\doteq\left(\begin{array}{c} x\\ y\\ t \end{array}\right)\mapsto X'\doteq\left(\begin{array}{c} x'\\ y'\\ t' \end{array}\right)=M\cdot\left(\begin{array}{c} x\\ y\\ t \end{array}\right) \] Quelle est la nouvelle écriture matricielle de $\mathcal{C}$ (à utiliser avec les $X'$) ?
    J'ai hélas deux interprétations de la question. J'imagine que la première est la bonne, car l'autre ne s'utilise pas avec $X'$.
    a) "Comment tester sur son image $X'$ si un $X$ tel que $X'=M\cdot X$ vérifie $X\in C$ ?"
    $\,^t (M^{-1}\cdot X')\cdot\boxed{C}\cdot(M^{-1}\cdot X')=0$, ou encore $\,^t X'\cdot(\,^t M^{-1}\cdot\boxed{C}\cdot M^{-1})\cdot X'=0$, d'où la nouvelle écriture $\,^t M^{-1}\cdot\boxed{C}\cdot M^{-1}$.
    b) "Comment tester si $X'\in C$ ?" $M.X \in C\Longleftrightarrow \,^t (M\cdot X)\cdot\boxed{C}\cdot(M\cdot X)=0$, ou encore $\,^t X\cdot(\,^t M\cdot\boxed{C}\cdot M)\cdot X=0$
    Début de la question : déjà traité ? Ensuite, bien qu'elle ne semble pas demandée, voici une matrice de colonnes propres (déjà donnée sous forme des six matrices dans le message précédent). $$\begin{pmatrix}16&4&1&8&4&8\\1&9&4&-3&-12&4\\25&1&16&-5&8&-40\\-4&6&-2&5&-1&-9\\5&3&-8&-8&10&6\\-20&2&4&-3&9&11\end{pmatrix}$$ Les lignes propres (de $H$, donc les lignes de $P^{-1}$) correspondaient aux valeurs $1,-\dfrac1 2, -1$, donc d'après la bath égalité \[ \widehat{h}\left(\tra{\Delta_{j}}\cdot\Delta_{k}\right)=\tra{M_{h}^{-1}}\cdot\tra{\Delta_{j}}\cdot\Delta_{k}\cdot M_{h}^{-1}=\left(\lambda_{j}^{-1}\lambda_{k}^{-1}\right)\;\left(\tra{\Delta_{j}}\cdot\Delta_{k}\right) \] les grandes colonnes propres devraient l'être pour les carrés suivis des produits : $1, \dfrac 1 4, 1, -\dfrac1 2, \dfrac1 2, -1$ ? Ah non, j'oublie de prendre les inverses de ces inverses, puisqu'on est revenu aux colonnes. Il s'agit donc des valeurs propres $1,4,1,-2,2,-1$.
    Quand l'aspect "progression géométrique" $\lambda_1\lambda_3=\lambda_2^2$ jouera un rôle, ça sera pour certaines colonnes, comme $1\times1=(-1)^2$, ou $1\times4=2^2$, ou $1\times4=(-2)^2$ ? Il me semble qu'on a perdu l'aspect "les six forment une suite géométrique", non ?
    Amicalement,
    Swingmustard

  • Modifié (March 2022)
    Bonjour !
    pldx1 a dit le 8 février
    2. On se donne une conique passant par $B,C$ et tangente à $AB,AC$. En barycentriques, l'équation de la conique est:  \[ \mathrm{eq}\left(A,B\right)\times\mathrm{eq}\left(A,C\right)+K\times\mathrm{eq}\left(BC\right)^{2}=Kx^{2}-yz \] On considère $U=tB+\left(1-t\right)C$. Un point $M$ de la droite $AU$ s'écrit $A+f\,U$. La condition pour que $M\left(f\right)$ soit sur la conique est $K-f^{2}\left(1-t\right)t$. Par ailleurs, on a  \[ \crra{A,U,M_{f},M_{g}}{}+1=\frac{f+g}{g} \] Or l'équation ci-dessus montre que les deux racines $f$et $g$ sont de somme nulle. Par conséquent, la division formée par $A,U$ et les deux intersections de $AU$ avec la conique est une division harmonique (que les intersections soient visibles ou non).la matrice directe agissant à gauche. As the joke says: les droites ne sont point des points, les points ne sont droite des droites. 
    Ne comprenant pas trop (n'y a-t-il pas un problème de signe dans la première égalité ?), j'ai pensé que l'occasion était bonne de remettre le couvert pour les coordonnées barycentriques.
    Suivant le conseil d'Eiden (Géométrie analytique classique), j'ai systématiquement inscrit le triangle de référence dans la conique.
    Page 51, Eiden dit aux débutants de mon espèce que l'équation du cercle circonscrit est simplement $$a^2yz+b^2zx+c^2xy=0$$ C'est la partie Cercle des dessins.

    Partie Ellipse : mes progrès récents mènent plus confortablement qu'il y a quelques mois aux coefficients $p$, $q$, $r$ de l'équation $$pyz+qzx+rxy=0$$ en partant d'une équation cartésienne et avec des matrices de passage.
    (Le premier dessin est un cas particulier : si le centre est sur l'un des côtés, il en est le milieu, et le coefficient correspondant est la somme des deux autres, comme Eiden le dit page 62.)

          
    Question (dont j'ignore la réponse).
    Existe-t-il un moyen "intrinsèque" d'accéder à $p$, $q$, $r$ : sans passer par une équation cartésienne ?
    Un angle à considérer ? Une intervention de l'excentricité ?
    Regardant la figure et comptant quelques carreaux, on dirait "Et hop, voilà, $p$, $q$, $r$, c'est tant et tant et tant."
    Le premier exemple laisse à penser que convoquer le centre de l'ellipse ne suffira pas, mais je suis prêt à pas mal de compromis, du moment qu'ils évitent l'équation cartésienne.
    Amicalement,
    Swingmustard
    P.S. Je suis bien conscient que pour les coniques du faisceau bitangent, le sommet $A$ du triangle devra quitter la conique, qui ne sera donc plus circonscrite. Mais la bitangence va nous assurer l'équation encore plus simple proposée par pldx1. À suivre.
  • Modifié (March 2022)
    Bonjour !
    Améliorons la formulation d'hier.
    Question1 : On se donne une ellipse circonscrite au triangle $ABC$, déterminer son équation barycentrique.
    Comme dirait pappus, encore faut-il que mon ellipse soit bien définie.
    Alors voilà, elle passe par $D(3:-3;-5)$ et par $E(9:6:-5)$.
    La théorie assure que la forme cherchée est $$p\,yz+q\,zx+r\,xy=0$$
    Les points $D$ et $E$ nous donnent deux équations.
    Or le triplet des coefficients cherchés est défini à un multiple près.
    Donc nous n'avons véritablement que deux inconnues, c'est plié.
    $$27\,yz+12\,zx+25\,xy=0$$
    Question 2 : La présentation des hypothèses est un chouia différente : on travaille sur un cas particulier.
    Les coordonnées barycentriques de $D$ et $E$ ne nous sont pas données.
    (Bien que cachées, elles sont identiques à celles de la question 1.)
    À la place, nous disposons d'une version dessinée, avec quadrillage.
    La question demeure "Déterminer l'équation barycentrique". 
    Nous pouvons procéder comme avant, mais il faudrait calculer les coordonnées barycentriques de $D$ et $E$.
    Et nous serions de retour à la question 1, or nous cherchons une autre méthode.
    Pour le cercle circonscrit, la formule $(p,q,r)=(a^2,b^2,c^2)$ utilise à fond le quadrillage et nous dispense merveilleusement du calcul des coordonnées de $D$ et $E$.
    Connaissez-vous une méthode pour obtenir le triplet $(p,q,r)$ de cette ellipse qui, faisant son miel du quadrillage, nous permettrait d'être presque aussi paresseux qu'avec l'équation du cercle ?
    Amicalement,
    Swingmustard
  • Bonjour,

    Comment être paresseux ? Facile. Il suffit d'amortir l'investissement sur un grand nombre d'exécutions. 
    Cela s'appelle créér une macro. Et donc, sous geogebra, on construit l'outil "tripolar" qui prend en entrée
    $A,B,C,D$ et donne en sortie $K_a,K_b,K_c$ et la droite qui les joint.

    Le code est: 


    \[ \begin{array}{cccc} 1 & \mathrm{Point} & A & given\;\\ 2 & \mathrm{Point} & B & given\;\\ 3 & \mathrm{Point} & C & given\;\\ 4 & \mathrm{Point} & D & given\;\\ 5 & \mathrm{Line} & ab & \mathrm{Line}[A,\;B]\;\\ 8 & \mathrm{Line} & bc & \mathrm{Line}[B,\;C]\;\\ 9 & \mathrm{Line} & ca & \mathrm{Line}[C,\;A]\;\\ 6 & \mathrm{Point} & D_{c} & \mathrm{Intersect}[\mathrm{Line}[D,\;C],\;ab]\;\\ 10 & \mathrm{Point} & D_{a} & \mathrm{Intersect}[\mathrm{Line}[D,\;A],\;bc]\;\\ 11 & \mathrm{Point} & D_{b} & \mathrm{Intersect}[\mathrm{Line}[D,\;B],\;ca]\;\\ 7 & \mathrm{Arc} & scC & Semicircle[C,\;D_{c}]\;\\ 12 & \mathrm{Arc} & scA & Semicircle[A,\;D_{a}]\;\\ 13 & \mathrm{Arc} & scB & Semicircle[B,\;D_{b}]\;\\ 14 & \mathrm{Point} & C_{D} & Reflect[D,\;scC]\;\\ 15 & \mathrm{Point} & B_{D} & Reflect[D,\;scB]\;\\ 16 & \mathrm{Point} & A_{D} & Reflect[D,\;scA]\;\\ 17 & \mathrm{Point} & K_{c} & \mathrm{Intersect}[\mathrm{Line}[D_{a},\;D_{b}],\;\mathrm{Line}[A_{D},\;B_{D}]]\;\\ 18 & \mathrm{Point} & K_{a} & \mathrm{Intersect}[\mathrm{Line}[D_{b},\;D_{c}],\;\mathrm{Line}[B_{D},\;C_{D}]]\;\\ 19 & \mathrm{Point} & K_{b} & \mathrm{Intersect}[\mathrm{Line}[D_{c},\;D_{a}],\;\mathrm{Line}[C_{D},\;A_{D}]]\;\\ 20 & \mathrm{Line} & f & \mathrm{Line}[K_{a},\;K_{b}]\; \end{array} \]

    Et alors, $tripolar(A,B,C,D)$ donne la tripolaire de $D$, tandis que $tripolar(A,B,C,E)$ donne la tripolaire de $E$. Les deux droites se coupent au point $P$ qui est le perspecteur de la conique $A,B,C,D,E$   wrt le triangle $A,B,C$. Cela se démontre (=yaka).  Et comme l'on a conservé la macro $conicir(A,B,C,P)$  qui trace la conique circonscrite de perspecteur $P$, yapuka.


    Cordialement, Pierre.
  • Modifié (March 2022)
    Bonjour !
    Merci pour cette construction, pldx1.

    j'ai demandé en vain qu'on éclaire ma lanterne sur "le perspecteur".
    Je suis donc toujours aussi ignorant sur cette notion.
    Grâce à toi, je conjecture au moins que celui d'une conique par rapport à un triangle inscrit est tout simplement
    le barycentre des trois sommets avec comme coefficients ... ceux de l'équation barycentrique de la conique !
    À vrai dire, je cherchais une autre forme de paresse.
    Une formule actuellement obtenue dépend de six déterminants, dont des combinaisons linéaires forment des produits de trois facteurs.
    Je crois que je la donnerai bientôt, en signe de reddition : le "miracle" du cercle ne se reproduit pas, c'est tout et ce n'est pas grave.
    Amicalement,
    Swingmustard
Connectez-vous ou Inscrivez-vous pour répondre.
Success message!